Patho Pharm II Exam 2

Pataasin ang iyong marka sa homework at exams ngayon gamit ang Quizwiz!

The nurse has received report that a client's bone marrow biopsy procedure was complicated with periods of bleeding. Which priority post-procedural intervention would the nurse delegate to the unlicensed assistive personnel on the team? Providing hydration Ice pack administration Vital sign measurement Skin inspection for ecchymosis

Ice pack administration

A client with hypertension asks the nurse what type of exercise she should do each day. What is the nurse's best response? "Exercise for an hour, but only three times a week." "Walk on the treadmill for 45 minutes every morning." "Begin walking and increase your distance as you can tolerate it." "Exercise only in the morning and stop when you get tired."

"Begin walking and increase your distance as you can tolerate it."

After you receive the change-of-shift report, which patient will you assess first? 1. 20-year-old with possible acute myelogenous leukemia who has just arrived on the medical unit 2. 38-year-old with aplastic anemia who needs teaching about decreasing infection risk before discharge 3. 40-year-old with lymphedema who requests help in putting on compression stockings before getting out of bed 4. 60-year-old with non-Hodgkin lymphoma who is refusing the prescribed chemotherapy regimen

1. 20-year-old with possible acute myelogenous leukemia who has just arrived on the medical unit

A group of patients is assigned to an RN-LPN/LVN team. The LPN/LVN should be assigned to provide patient care and administer medications to which patient? 1. 36-year-old with chronic kidney failure who will need a subcutaneous injection of epoetin alfa (Procrit) 2. 39-year-old with hemophilia B who has been admitted to receive a transfusion of PRBCs 3. 50-year-old with newly diagnosed polycythemia vera who will require phlebotomy 4. 55-year-old with a history of stem cell transplantation who has a bone marrow aspiration scheduled

1. 36-year-old with chronic kidney failure who will need a subcutaneous injection of epoetin alfa (Procrit)

You are working with an experienced UAP and an LPN/LVN on the telemetry unit. A client who had an acute MI 3 days ago has a nursing diagnosis of Activity Intolerance related to fatigue and chest pain. Which nursing activity included in the care plan is best delegated to the LPN/LVN? 1. Administering nitroglycerin (Nitrostat) if chest discomfort occurs during client activities 2. Monitoring pulse, blood pressure, and oxygen saturation before and after client ambulation 3. Teaching the client energy conservation techniques to decrease myocardial oxygen demand 4. Explaining the rationale for alternating rest periods with exercise to the client and family

1. Administering nitroglycerin (Nitrostat) if chest discomfort occurs during client activities

The health care provider prescribes these actions for a client who was admitted with acute substernal chest pain. Which actions are appropriate to delegate to an experienced LPN/LVN who is working with you in the ED? (Select all that apply.) 1. Attaching cardiac monitor leads 2. Giving heparin 5000 units IV push 3. Administering morphine sulfate 4 mg IV 4. Obtaining a 12-lead electrocardiogram (ECG) 5. Asking the client about pertinent medical history 6. Having the client chew and swallow aspirin 162 mg

1. Attaching cardiac monitor leads 4. Obtaining a 12-lead electrocardiogram (ECG) 6. Having the client chew and swallow aspirin 162 mg

Ventricular fibrillation is identified in an unresponsive 50-year-old client who has just arrived in the ED. Which action will you take first? 1. Defibrillate at 200 J. 2. Start cardiopulmonary resuscitation (CPR). 3. Administer epinephrine (Adrenalin) 1 mg IV. 4. Intubate and manually ventilate.

1. Defibrillate at 200 J.

You are working in an outpatient clinic where many vascular diagnostic tests are performed. Which task associated with vascular testing is most appropriate to delegate to an experienced UAP? 1. Measuring ankle and brachial pressures in a client for whom the ankle-brachial index is to be calculated 2. Checking blood pressure and pulse every 10 minutes in a client who is undergoing exercise testing 3. Obtaining information about allergies from a client who is scheduled for left leg contrast venography 4. Providing brief client teaching for a client who will undergo a right subclavian vein Doppler study

1. Measuring ankle and brachial pressures in a client for whom the ankle-brachial index is to be calculated

These activities are included in the care plan for a 78-year-old patient admitted to the hospital with anemia caused by possible gastrointestinal bleeding. Which activity can you delegate to an experienced UAP? 1. Obtaining stool specimens for fecal blood test (Hemoccult) slides 2. Having the patient sign a colonoscopy consent form 3. Giving the prescribed polyethylene glycol electrolyte solution (GoLYTELY) 4. Checking for allergies to contrast dye or shellfish

1. Obtaining stool specimens for fecal blood test (Hemoccult) slides

You are making a room assignment for a newly arrived patient whose laboratory test results indicate pancytopenia. Which patient will be the best roommate for the new patient? 1. Patient with digoxin toxicity 2. Patient with viral pneumonia 3. Patient with shingles 4. Patient with cellulitis

1. Patient with digoxin toxicity

While admitting a client, you obtain this information about her cardiovascular risk factors: Her mother and two siblings have had myocardial infarctions (MIs). The client smokes and has a 20 pack-year history of cigarette use. Her work as a mail carrier involves a lot of walking. She takes metoprolol (Lopressor) for hypertension, and her blood pressure has been in the range of 130/60 to 138/85 mm Hg. Which interventions will be important to include in the discharge plan for this client? (Select all that apply.) 1. Referral to community programs that assist in smoking cessation 2. Teaching about the impact of family history on cardiovascular risk 3. Education about the need for a change in antihypertensive therapy 4. Assistance in reducing the stress associated with her cardiovascular risk 5. Discussion of the risks associated with having a sedentary lifestyle

1. Referral to community programs that assist in smoking cessation 2. Teaching about the impact of family history on cardiovascular risk

A patient with acute myelogenous leukemia is receiving induction-phase chemotherapy. Which assessment finding requires the most rapid action? 1. Serum potassium level of 7.8 mEq/L 2. Urine output less than intake by 400 mL 3. Inflammation and redness of the oral mucosa 4. Ecchymoses present on the anterior trunk

1. Serum potassium level of 7.8 mEq/L

The nurse in the outpatient clinic is assessing a 22-year-old who required a splenectomy after a recent motor vehicle accident. Which information obtained during the assessment will be of most immediate concern to the nurse? 1. The patient engages in unprotected sex. 2. The oral temperature is 100° F (37.8° C). 3. There is abdominal pain with light palpation. 4. The patient admits to occasional marijuana use.

2. The oral temperature is 100° F (37.8° C).

You are caring for a hospitalized client with heart failure who is receiving captopril (Capoten) and spironolactone (Aldactone). Which laboratory value will be most important to monitor? 1. Sodium level 2. Blood urea nitrogen level 3. Potassium level 4. Alkaline phosphatase level

3. Potassium level

Two weeks ago, a 63-year-old client with heart failure received a new prescription for carvedilol (Coreg) 3.125 mg orally. When evaluating the client in the cardiology clinic, you obtain the following data. Which finding is of most concern? 1. Reports of increased fatigue and activity intolerance 2. Weight increase of 0.5 kg over a 1-week period 3. Sinus bradycardia at a rate of 48 beats/min 4. Traces of edema noted over both ankles

3. Sinus bradycardia at a rate of 48 beats/min

You have given morphine sulfate 4 mg IV to a client who has an acute MI. When you evaluate the client's response 5 minutes after giving the medication, which finding indicates a need for immediate further action? 1. Blood pressure decrease from 114/65 to 106/58 mm Hg 2. Respiratory rate drop from 18 to 12 breaths/min 3. Cardiac monitor indicating sinus rhythm at a rate of 96 beats/min 4. Persisting chest pain at a level of 1 (on a scale of 0 to 10)

4. Persisting chest pain at a level of 1 (on a scale of 0 to 10)

You obtain the following data about a patient admitted with multiple myeloma. Which information has the most immediate implications for the patient's care? 1. The patient reports chronic bone pain. 2. The blood uric acid level is very elevated. 3. The 24-hour urine test shows Bence Jones proteins. 4. The patient reports new-onset leg numbness.

4. The patient reports new-onset leg numbness.

You are transferring a patient with newly-diagnosed chronic myeloid leukemia to an LTC facility. Which information is most important to communicate to the LTC charge nurse before transferring the patient? 1. Philadelphia chromosome is present in the patient's blood smear. 2. Glucose level is elevated as a result of prednisone (Deltasone) therapy. 3. There has been a 20-lb weight loss over the last year. 4. The patient's chemotherapy has resulted in neutropenia.

4. The patient's chemotherapy has resulted in neutropenia.

You are preparing to administer the following medications to a client with multiple health problems who has been hospitalized with deep vein thrombosis. Which medication is most important to double-check with another licensed nurse? 1. Famotidine (Pepcid) 20 mg IV 2. Furosemide (Lasix) 40 mg IV 3. Digoxin (Lanoxin) 0.25 mg PO 4. Warfarin (Coumadin) 2.5 mg PO

4. Warfarin (Coumadin) 2.5 mg PO

A client complains of chest pain during inhalation. The nurse hears a rubbing friction sound when auscultating the breath sounds. What is the nurse's interpretation of this information? A pleural friction rub is present. The client has angina with tactile fremitus. Fluid is accumulating in the client's lungs. Bronchovesicular breath sounds are present.

A pleural friction rub is present.

The nurse is teaching a client about the use of transdermal nitroglycerin ointment. Which site should be used for the most effective absorption? On the chest near the heart Area on the chest without hair Muscular area over thigh Upper abdominal area

Area on the chest without hair

What will caring for a patient with a diagnosis of polycythemia vera likely require the nurse to do? Encourage deep breathing and coughing. Assist with or perform phlebotomy at the bedside. Teach the patient how to maintain a low-activity lifestyle. Perform thorough and regularly scheduled neurologic assessments.

Assist with or perform phlebotomy at the bedside.

A client has a significant family history of coronary artery disease (CAD) and history of exercise-induced asthma. Which medication would the nurse anticipate would treat a modifiable risk factor of CAD? Fluticasone/salmeterol Budesonide Atorvastatin Amlodipine

Atorvastatin

A client has been receiving filgrastim. What side effect might the nurse anticipate that the client may experience? Fatigue and malaise Bone pain Bruising Nausea and vomiting

Bone pain

The nurse is assessing an adult female client who is 65-years-old for risk factors associated with atherosclerosis. Which factors are nonmodifiable risk factors? Elevated BMI Hypertension Elevated cholesterol Older adult

Older adult

Where on the chest would the nurse place the stethoscope to evaluate a client's apical heart rate? On the left side, midaxillary level On the sternum, directly over the heart On the left side at the midclavicular line, fifth intercostal space On the right side at the midclavicular line, fourth intercostal space

On the left side at the midclavicular line, fifth intercostal space

The nurse admitting a cardiac client is performing the admitting physical assessment. What action by the nurse could cause a detrimental client response? Positioning the client in semi-Fowler's position and asking the client to lean forward to listen for heart sounds Positioning the client in the left lateral position to determine placement of the point of maximum impulse (PMI) Palpating both carotid arteries at the same time to compare quality and character Asking the client to flex his or her legs and press feet against the nurse's hands to determine strength of extremities

Palpating both carotid arteries at the same time to compare quality and character

The nurse is completing a focused vascular assessment on a client. Which intervention should be avoided? Palpating both carotids at the same time Obtaining peripheral pulses on both extremities Measuring the blood pressure in both arms Assessing capillary refill

Palpating both carotids at the same time

What would be the most important information for the nurse to document as an indication of the progress of a client with heart failure (HF)? Telemetry recordings Accurate intake record Length of afternoon nap Record of daily weight loss

Record of daily weight loss

How will the nurse assist the client with atherosclerotic heart disease (ASHD) to make effective changes in his or her diet? Review favorite foods and recipes with the client and help identify high fat content; substitute monounsaturated oils. Provide the client with a list of foods with high fat content that are to be avoided and how low-cholesterol foods can be substituted. Discuss with the client that foods containing sodium and saturated fats should be avoided in the diet. Review with the client the impact a diet with increased sodium and saturated fats has on the development of atherosclerotic heart disease.

Review favorite foods and recipes with the client and help identify high fat content; substitute monounsaturated oils.

The nurse is completing the initial physical assessment on a newly admitted older adult client. The nurse determines the cardiac point of maximum impulse (PMI) is at the sixth intercostal space and lateral to the midclavicular line. What is the best nursing action based on this information? Determine if the client has increased risk factors for coronary artery disease. Document the location of the PMI and continue with the assessment. Review the client's history for presence of hypertension. Perform a neurologic assessment for evidence of a stroke.

Review the client's history for presence of hypertension.

The nurse anticipates administering a beta-blocker to a patient experiencing which dysrhythmia? Sinus Bradycardia V-Fib Sinus Tachycardia Normal Sinus Rhythm

Sinus Tachycardia

A client is at risk for hypovolemic shock. Which assessment finding is an early indication of hypovolemic shock? Bradypnea Complete heart block Tachycardia Low blood pressure

Tachycardia

You are preparing to implement teaching about a heart-healthy diet and activity levels for a client who has had an MI and her husband. The client says, "I don't see why I need any teaching. I don't think I need to change anything right now." Which response is most appropriate? 1. "Do you think your family may want you to make some lifestyle changes?" 2. "Can you tell me why you don't feel that you need to make any changes?" 3. "You are still in the stage of denial, but you will want this information later on." 4. "Even though you don't want to change, it's important that you have this teaching."

2. "Can you tell me why you don't feel that you need to make any changes?"

A client seen in the clinic with shortness of breath and fatigue is being evaluated for a possible diagnosis of heart failure. Which laboratory result will be most useful to monitor? 1. Serum potassium 2. B-type natriuretic peptide 3. Blood urea nitrogen 4. Hematocrit

2. B-type natriuretic peptide

You are working in the ED caring for a client who was just admitted with left anterior chest pain, possible ACS. Which action will you take first? 1. Insert an IV catheter. 2. Auscultate heart sounds. 3. Administer sublingual nitroglycerin. 4. Draw blood for troponin I measurement.

3. Administer sublingual nitroglycerin.

An 80-year-old client on the coronary step-down unit tells you that he does not want to take the ordered docusate (Colace) because he does not have any problems with constipation. Which action is most appropriate? 1. Document the medication on the client's chart as "refused." 2. Mix the medication with food and administer it to the client. 3. Explain that his decreased activity level may cause constipation. 4. Reinforce that the docusate has been prescribed for a good reason.

3. Explain that his decreased activity level may cause constipation.

A client with chronic renal disease receiving recombinant epoetin alfa is most likely diagnosed with which of the following conditions? Anemia Hypertension Neutropenia Thrombocytopenia

Anemia

While providing hygiene for a cardiac client, the nurse observes an episode of ventricular tachycardia for about 10 seconds; the client's rhythm spontaneously returns to the previous rate of 110 beats/min. What would be the best nursing action? Notify the health care provider immediately. Begin oxygen and administer an amiodarone intravenous loading dose. Prepare for immediate elective cardioversion. Administer atropine via IV push.

Begin oxygen and administer an amiodarone intravenous loading dose.

The nurse should assess the client receiving heparin for which common untoward effect? Generalized dermatitis Hematuria Urinary retention Vitamin K deficiency

Hematuria

The nurse would anticipate which laboratory test would be used in the diagnosis of pernicious anemia? Homocysteine D-dimer Ferritin Platelet count

Homocysteine

A client has received 3 units of packed red blood cells over the past 24 hours. What laboratory finding would the nurse need to report to the healthcare provider? Hypokalemia Hyponatremia Hyperkalemia Hypernatremia

Hyperkalemia

The nurse would allow a client taking which of the following medications to have grapefruit juice on the breakfast tray? Iron nifedipine lovastatin buspirone

Iron

A client is beginning treatment with warfarin. What would the nurse teach the client regarding diet? Decrease whole grains and oats. Avoid milk and milk products. Increase intake of eggs and red meat. Keep a constant intake of leafy, green vegetables.

Keep a constant intake of leafy, green vegetables.

A client diagnosed with a T5 spinal cord injury has a blood pressure (BP) of 100/68 mm Hg and a pulse of 68 beats/min. Which type of shock is the client experiencing? Anaphylactic Hypovolemic Neurogenic Cardiogenic

Neurogenic

A chemotherapy patient receiving filgrastim is most likely diagnosed with which of the following conditions? Anemia Hypertension Neutropenia Thrombocytopenia

Neutropenia

A client had an aortic-femoral bypass graft. The nurse assists the client back to bed after he has ambulated. What is most important for the nurse to assess? Blood pressure in both arms Radial pulse rate and quality Temperature of the affected extremity Pedal pulse of the affected extremity

Pedal pulse of the affected extremity

Teaching the patient with rheumatic fever about the disease, the nurse explains that rheumatic fever is a Streptococcus viridans infection. a viral infection of endocardium and valves. a sequela of group A streptococcal infection. frequently triggered by immunosuppressive therapy.

a sequela of group A streptococcal infection.

The nurse instructs the patient to take the prescribed diuretic at what time of day? at breakfast at lunch at dinner at bedtime

at breakfast

Conservative management has been unsuccessful for a patient diagnosed with Raynaud's phenomenon. Which treatment does the nurse anticipate the health care provider will prescribe next for this​ patient? transluminal balloon angioplasty amputation of the affected digits peripheral arterial bypass surgery calcium channel blockers

calcium channel blockers

The nurse suspects the presence of a deep vein thrombosis based on which findings? paresthesia and coolness of the leg. pain in the calf that occurs with exercise. generalized edema of the involved extremity. pallor and cyanosis of the involved extremity.

generalized edema of the involved extremity.

The nurse knows intermittent claudication, the classic symptom of lower extremity peripheral arterial disease (PAD), is caused by which pathophysiology problem? inadequate blood flow to the beginning of a venous leg ulcer. inadequate blood flow to the nerves of the feet. inadequate blood flow to the leg muscles during exercise. inadequate blood flow to the skin after application of the heat.

inadequate blood flow to the leg muscles during exercise.

The clinic nurse caring for a group of essential hypertension patients understands this disease process is influenced by which contributing factors? Select all that apply. increased SNS activity increased sodium-retaining hormones decreased body weight decreased sodium intake excessive alcohol intake

increased SNS activity increased sodium-retaining hormones excessive alcohol intake

The nurse is preparing to administer spironolactone (Aldactone) to a hypertensive patient. Which finding in the patient's medical record should be reported to the physician immediately? abdominal cramping, diarrhea, vomiting potassium level sinus bradycardia blood pressure

potassium level

A patient is being prepared for an abdominal aortic aneurysm repair. The nurse suspects rupture of the aneurysm into the retroperitoneal space when observing which clinical manifestation(s)? the patient becomes dizzy and short of breath. the patient complains of sudden, severe back pain. a bruit and thrill are present at the site of the aneurysm. the patient develops blue, patchy mottling of the feet and toes.

the patient complains of sudden, severe back pain.

The most common cause of myocarditis is viruses. radiation. endocarditis. myocardial infarction.

viruses.

A client has stage 2 hypertension. What is the most common symptom reported at this stage of hypertension? Epistaxis Blurred vision Dyspnea on exertion No symptoms

No symptoms

The nurse finds the patient pale, cool, weak, confused, and short of breath. Which medication would the nurse anticipate administering to this patient? Atropine Lidocaine Digoxin Metoprolol

Atropine

The nurse is assessing a client with peripheral arterial disease. What would the nurse expect to find on assessment of this client? Thin, fragile toenails Dependent rubor Bounding arterial pulses Warm, erythematous legs

Dependent rubor

The nurse would anticipate which common serious acute complication of polycythemia vera? Heart failure Pulmonary edema Stroke Renal failure

Stroke

A chemotherapy patient receiving oprelvekin is most likely diagnosed with which of the following conditions? Anemia Hypertension Neutropenia Thrombocytopenia

Thrombocytopenia

The nurse caring for a patient scheduled for a percutaneous transluminal balloon valvuloplasty understands that this procedure is the treatment of choice for combined aortic stenosis and aortic regurgitation. involves the insertion of a transventricular dilator into the opening of the valve. is recommended for patients who are poor candidates for more extensive valvular surgery. is a last resort treatment when other valvular repair procedures have not been effective.

is recommended for patients who are poor candidates for more extensive valvular surgery.

The nurse is assigned to care for a client newly diagnosed with septic shock. Which laboratory finding is consistent with this diagnosis? Decreased potassium and sodium Decreased creatinine and BUN Elevated prothrombin level Elevated lactate level

Elevated lactate level

At 10:00 am, a hospitalized client receives a new order for transesophageal echocardiography (TEE) as soon as possible. Which action will you take first? 1. Put the client on "nothing by mouth" (NPO) status. 2. Teach the client about the procedure. 3. Insert an IV catheter in the client's forearm. 4. Attach the client to a cardiac monitor.

1. Put the client on "nothing by mouth" (NPO) status.

The nurse would question which medication order for a client who is receiving heparin? Cortisone Aspirin Glipizide Digoxin

Aspirin

A client with a diagnosis of chronic heart failure is taking furosemide and digoxin. To decrease the risk of digoxin toxicity, which action should be performed by the nurse? Administer digoxin for a heart rate of 68 beats/min. Assess for diarrhea, diaphoresis, and anorexia. Instruct the client to eat foods low in potassium. Give a digoxin dose for a serum potassium of 4.2 mEq/L (4.2 mmol/L).

Assess for diarrhea, diaphoresis, and anorexia.

A 62-year-old woman weighs 92 kg and has a history of daily alcohol intake, smoking, high blood pressure, high sodium intake, and sedentary lifestyle. The nurse identifies which risk factors as most highly related to peripheral arterial disease? sex and age. weight and alcohol intake. cigarette smoking and hypertension. sedentary lifestyle and high sodium intake.

cigarette smoking and hypertension.

The nurse recognizes which adverse reaction associated with ACE inhibitors is the most common and typically leads to disruption of therapy? constipation cough sexual dysfunction tachycardia

cough

A client has been receiving chlorothiazide and lisinopril and is admitted to the unit with a complaint of severe headache and a blood pressure (BP) of 222/112 mm Hg, a pulse of 98 beats/min, and respirations of 26 breaths/min. What would be a priority assessment question? "Have you taken your medications every day?" "How many nonsteroidal antiinflammatory drugs (NSAIDs) did you take today?" "Have you been eating any grapefruits or other citrus fruits?" "Have there been any stressful events recently?"

"Have you taken your medications every day?"

The nurse caring for a group of hypertensive patients teaches the group strategies to modify their risk factors. Which statement, made by one of the patients, requires further teaching related to modification of risk factors? "I will stop smoking as nicotine raises my blood pressure." "I need to keep my blood glucose under better control." "I am going to investigate my family's history of high blood pressure." "I have decided to lose at least 22 lbs to decrease my blood pressure."

"I am going to investigate my family's history of high blood pressure."

A 45-year-old male reports to the clinic for a follow-up visit after being prescribed atenolol. Which statement made by the client explains the reason why he does not want to take the atenolol? "It makes me feel hungry all the time." "I feel tired and lethargic." "I have problems falling and staying asleep." "I have a decreased sex drive."

"I have a decreased sex drive."

A client is being discharged after implantation of a permanent pacemaker. What comment by the client would indicate to the nurse the need for additional teaching? "I know I will have to stay away from microwaves because they interfere with the pacemaker." "I won't raise my arm on the side of the pacemaker until I go back for my office visit." "I can fly and travel anywhere I want to go; the pacemaker will keep on working." "I know I will have to have regular checkups to make sure the pacemaker is functioning properly."

"I know I will have to stay away from microwaves because they interfere with the pacemaker."

Which comment by the client would indicate to the nurse the client understands how to take his sublingual nitroglycerin? "I will call 911 if the pain does not go away after taking three doses of the medication 5 minutes apart." "I will put the nitroglycerin between my cheek and gum whenever I begin to have any chest pain." "I will take the nitroglycerin when I get up in the morning and whenever I experience chest pain during the day." "I know that nitroglycerin is important to help prevent any further damage to the arteries in my heart."

"I will call 911 if the pain does not go away after taking three doses of the medication 5 minutes apart."

A client with severe atherosclerotic disease and hypertension has been discharged from the hospital. What comment by the client would indicate achievement of an effective client outcome? "I need to exercise just once a week on the treadmill." "I will check my blood pressure daily." "When I finish my new simvastatin medication, I do not need to take it any longer." "I need to adjust my diet by including more red meats."

"I will check my blood pressure daily."

The nurse is assisting a client who is being discharged and is given a prescription for metoprolol. Which statement by the client indicates she understands how to take her medication? "I will need to stand up gradually because the metoprolol may make me a little dizzy at first." "I should have relief from my shortness of breath when I take the metoprolol." "I don't have to worry about my diabetic medicine; metoprolol will not affect it." "I should be able to decrease my dose of metoprolol when I begin to feel better."

"I will need to stand up gradually because the metoprolol may make me a little dizzy at first."

The nurse determines a nursing problem related to ineffective peripheral perfusion related to decreased arterial blood flow for a client with diabetes who had been diagnosed with peripheral arterial disease. The nurse would identify the need for further teaching based on what client statement? "I will make sure my shoes fit well and do not rub any blisters." "I will not sit in one position for longer than an hour." "I will soak my feet in hot water to increase the circulation." "I will walk every day and stop when I begin to have pain in my legs."

"I will soak my feet in hot water to increase the circulation."

Which statement by the client indicates that he understands how to take sublingual nitroglycerin (NTG)? "If I have a chest pain, I'll immediately stop what I am doing, sit down, and take the medication." "I'll chew one tablet, then let it dissolve in my mouth if the chest pain lasts more than 5 minutes." "I'll take one dose every 15 minutes for a total of three doses. If the pain doesn't stop, I'll call the doctor." "If I have chest pain, I'll call the doctor and put two tablets under my tongue."

"If I have a chest pain, I'll immediately stop what I am doing, sit down, and take the medication."

A 26-year-old patient with early stage Hodgkin's lymphoma asks the nurse, "How long do I have to live?" The nurse's best response to the patient is "Since no one can predict how long someone will live, try to focus on the present." "It will depend on how your disease responds to radiation, but most patients do well." "With ongoing maintenance chemotherapy, the 10-year survival rate is very good." "Most patients with your stage of Hodgkin's disease are treated successfully."

"Most patients with your stage of Hodgkin's disease are treated successfully."

The nurse is teaching a client with hypertension about the prescribed antihypertensive medications: hydrochlorothiazide (HCTZ) and enalapril. What is important to include in this teaching? "Stand up slowly to decrease problems with dizziness." "Increase fluid intake because of increased loss of body fluids." "When you begin to feel better, the doctor will decrease your medications." "Stay out of the sunshine and make sure you have adequate sodium intake."

"Stand up slowly to decrease problems with dizziness."

What are appropriate nursing questions when admitting an adult female client who has the diagnosis of thrombocytopenic purpura (TP)? Select all that apply. "When did your headaches start?" "What medications do you take?" "How heavy are your menstrual periods?" "Do you take any herbal supplements?" "Have you noticed a skin rash recently?" "Do you have a history of epilepsy?"

"What medications do you take?" "How heavy are your menstrual periods?" "Do you take any herbal supplements?" "Have you noticed a skin rash recently?"

As a home health nurse, you are obtaining an admission history for a patient who has deep vein thrombosis and is taking warfarin (Coumadin) 2 mg daily. Which statement by the patient is the best indicator that additional teaching about warfarin may be needed? 1. "I have started to eat more healthy foods like green salads and fruit." 2. "The doctor said that it is important to avoid becoming constipated." 3. "Coumadin makes me feel a little nauseated unless I take it with food." 4. "I will need to have some blood testing done once or twice a week."

1. "I have started to eat more healthy foods like green salads and fruit."

During the initial postoperative assessment of a client who has just been transferred to the postanesthesia care unit after repair of an abdominal aortic aneurysm, you obtain these data. Which finding has the most immediate implications for the client's care? 1. Arterial line indicates a blood pressure of 190/112 mm Hg. 2. Cardiac monitor shows frequent premature atrial contractions. 3. There is no response to verbal stimulation. 4. Urine output is 40 mL of amber urine.

1. Arterial line indicates a blood pressure of 190/112 mm Hg.

A 50-year-old male was referred to the clinic for evaluation of blood pressure. Which blood pressure readings, taken at separate visits over a two week period, confirm the diagnosis of stage 2 hypertension? 110/66 and 120/78 144/94 and 120/76 112/78 and 118/68 140/90 and 160/96

140/90 and 160/96

While working on the cardiac step-down unit, you are serving as preceptor to a newly graduated RN who has been in a 6-week orientation program. Which client will be best to assign to the new graduate? 1. 19-year-old with rheumatic fever who needs discharge teaching before going home with a roommate today 2. 33-year-old admitted a week ago with endocarditis who will be receiving ceftriaxone (Rocephin) 2 g IV 3. 50-year-old with newly diagnosed stable angina who has many questions about medications and nursing care 4. 75-year-old who has just been transferred to the unit after undergoing coronary artery bypass grafting yesterday

2. 33-year-old admitted a week ago with endocarditis who will be receiving ceftriaxone (Rocephin) 2 g IV

You have just received a change-of-shift report about these clients on the coronary step-down unit. Which one will you assess first? 1. 26-year-old with heart failure caused by congenital mitral stenosis who is scheduled for balloon valvuloplasty later today 2. 45-year-old with constrictive cardiomyopathy who developed acute dyspnea and agitation about 1 hour before the shift change 3. 56-year-old who underwent coronary angioplasty and stent placement yesterday and has reported occasional chest pain since the procedure 4. 77-year-old who was transferred from the intensive care unit 2 days ago after coronary artery bypass grafting and has a temperature of 100.6° F (38.1° C)

2. 45-year-old with constrictive cardiomyopathy who developed acute dyspnea and agitation about 1 hour before the shift change

A 32-year-old patient with sickle cell anemia is admitted to the hospital during a sickle cell crisis. Which action prescribed by the health care provider will you implement first? 1. Give morphine sulfate 4 to 8 mg IV every hour as needed. 2. Administer 100% oxygen using a nonrebreather mask. 3. Start a 14-gauge IV line and infuse normal saline at 200 mL/hr. 4. Give pneumococcal (Pneumovax) and Haemophilus influenzae (ActHIB) vaccines.

2. Administer 100% oxygen using a nonrebreather mask.

A patient in a long-term care (LTC) facility who has chronic lymphocytic leukemia has a nursing diagnosis of Activity Intolerance related to weakness and anemia. Which nursing activity will you delegate to the UAP? 1. Evaluating the patient's response to normal activities of daily living 2. Checking the patient's blood pressure and pulse rate after ambulation 3. Determining which self-care activities the patient can do independently 4. Assisting the patient in choosing a diet that will improve strength

2. Checking the patient's blood pressure and pulse rate after ambulation

You are the charge nurse for the coronary care step-down unit. Which client is best to assign to a float RN who has come for the day from the general medical-surgical unit? 1. Client requiring discharge teaching about coronary artery stenting before going home today 2. Client receiving IV furosemide (Lasix) to treat acute left ventricular failure 3. Client who just transferred in from the radiology department after a coronary angioplasty 4. Client just admitted with unstable angina who has orders for a heparin infusion and aspirin

2. Client receiving IV furosemide (Lasix) to treat acute left ventricular failure

As the charge nurse in a long-term care facility that employs RNs, LPNs/LVNs, and UAPs, you have developed a plan for the ongoing assessment of all residents with a diagnosis of heart failure. Which activity included in the plan is most appropriate to delegate to an LPN/LVN team member? 1. Weighing all residents with heart failure each morning 2. Listening to lung sounds and checking for edema each week 3. Reviewing all heart failure medications with residents every month 4. Updating activity plans for residents with heart failure every quarter

2. Listening to lung sounds and checking for edema each week

A patient who has been receiving cyclosporine (Sandimmune) following an organ transplantation is experiencing the following symptoms. Which one is of most concern? 1. Bleeding of the gums while brushing the teeth 2. Nontender lump in the right groin 3. Occasional nausea after taking the medication 4. Numbness and tingling of the feet

2. Nontender lump in the right groin

The health care provider telephones you with new prescriptions for a client with unstable angina who is already taking clopidogrel (Plavix). Which medication is most important to clarify further with the health care provider? 1. Aspirin (Ecotrin) 162 mg daily 2. Omeprazole (Prilosec) 20 mg daily 3. Metoprolol (Lopressor) 50 mg daily 4. Nitroglycerin patch (Nitrodur) 0.4 mg/hr

2. Omeprazole (Prilosec) 20 mg daily

After a car accident, a patient with a medical alert bracelet indicating hemophilia A is admitted to the emergency department (ED). Which action prescribed by the health care provider will you implement first? 1. Transport to the radiology department for cervical spine radiography. 2. Transfuse factor VII concentrate. 3. Type and cross-match for 4 units of packed red blood cells (PRBCs). 4. Infuse normal saline at 250 mL/hr.

2. Transfuse factor VII concentrate.

You are working in the emergency department (ED) when a client arrives reporting substernal and left arm discomfort that has been going on for about 3 hours. Which laboratory test will be most useful in determining whether you should anticipate implementing the acute coronary syndrome (ACS) standard protocol? 1. Creatine kinase MB level 2. Troponin I level 3. Myoglobin level 4. C-reactive protein level

2. Troponin I level

A 67-year-old who is receiving chemotherapy for lung cancer is admitted to the hospital with thrombocytopenia. Which statement made by the patient when you are obtaining the admission history is of most concern? 1. "I've noticed that I bruise more easily since the chemotherapy started." 2. "My bowel movements are soft and dark brown." 3. "I take one aspirin every morning because of my history of angina." 4. "My appetite has decreased since the chemotherapy started."

3. "I take one aspirin every morning because of my history of angina."

A 22-year-old with stage I Hodgkin disease is admitted to the oncology unit for radiation therapy. During the initial assessment, the patient tells you, "Sometimes I'm afraid of dying." Which response is most appropriate at this time? 1. "Many individuals with this diagnosis have some fears." 2. "Perhaps you should ask the doctor about medication." 3. "Tell me a little bit more about your fear of dying." 4. "Most people with stage I Hodgkin disease survive."

3. "Tell me a little bit more about your fear of dying."

As charge nurse, you are making the daily assignments on the medical-surgical unit. Which patient is best assigned to a float nurse who has come from the postanesthesia care unit (PACU)? 1. 30-year-old patient with thalassemia major who has an order for subcutaneous infusion of deferoxamine (Desferal) 2. 43-year-old patient with multiple myeloma who requires discharge teaching 3. 52-year-old patient with chronic gastrointestinal bleeding who has returned to the unit after a colonoscopy 4. 65-year-old patient with pernicious anemia who has just been admitted to the unit

3. 52-year-old patient with chronic gastrointestinal bleeding who has returned to the unit after a colonoscopy

You make a home visit to evaluate a hypertensive client who has been taking enalapril (Vasotec). Which finding indicates that you need to contact the health care provider about a change in the drug therapy? 1. Client reports frequent urination. 2. Client's blood pressure is 138/86 mm Hg. 3. Client coughs often during the visit. 4. Client says, "I get dizzy sometimes."

3. Client coughs often during the visit.

You are monitoring the cardiac rhythms of clients in the coronary care unit. Which client will need immediate intervention? 1. Client admitted with heart failure who has atrial fibrillation with a rate of 88 beats/min while at rest 2. Client with a newly implanted demand ventricular pacemaker who has occasional periods of sinus rhythm at a rate of 90 to 100 beats/min 3. Client who has just arrived on the unit with an acute MI and has sinus rhythm at a rate of 76 beats/min with frequent premature ventricular contractions 4. Client who recently started taking atenolol (Tenormin) and has a first-degree heart block, with a rate of 58 beats/min

3. Client who has just arrived on the unit with an acute MI and has sinus rhythm at a rate of 76 beats/min with frequent premature ventricular contractions

You are developing a standardized care plan for the postoperative care of clients undergoing cardiac surgery. Which nursing activity included in the care plan will need to be performed by RN staff members? 1. Removing chest and leg dressings on the second postoperative day and cleaning the incisions with antibacterial swabs 2. Reinforcing client and family teaching about the need to deep breathe and cough at least every 2 hours while awake 3. Developing an individual plan for discharge teaching based on discharge medications and needed lifestyle changes 4. Administering oral analgesic medications as needed before helping the client out of bed on the first postoperative day

3. Developing an individual plan for discharge teaching based on discharge medications and needed lifestyle changes

A patient with graft-versus-host disease (GVHD) after bone marrow transplantation is being cared for on the medical unit. Which nursing activity is best delegated to a travel RN? 1. Administering oral cyclosporine (Sandimmune) to the patient 2. Assessing the patient for signs of infection caused by GVHD 3. Infusing 5% dextrose in 0.45% saline at 125 mL/hr into the patient 4. Educating the patient about ways to prevent and detect infection

3. Infusing 5% dextrose in 0.45% saline at 125 mL/hr into the patient

You are the charge nurse on an oncology unit when a patient with an absolute neutrophil count of 300/μL is admitted. Which staff member should you assign to provide care for this patient, under the supervision of an experienced oncology RN? 1. LPN/LVN who has floated from the same-day surgery unit 2. RN from a staffing agency who is being oriented to the oncology unit 3. LPN/LVN with 2 years of experience on the oncology unit 4. RN who recently transferred to the oncology unit from the ED

3. LPN/LVN with 2 years of experience on the oncology unit

A client who has endocarditis with vegetation on the mitral valve suddenly reports severe left foot pain. You note that no pulse is palpable in the left foot and that it is cold and pale. Which action should you take next? 1. Lower the client's left foot below heart level. 2. Administer oxygen at 4 L/min to the client. 3. Notify the client's physician about the change in status. 4. Reassure the client that embolization is common in endocarditis.

3. Notify the client's physician about the change in status.

You are providing orientation for a new RN who is preparing to administer packed red blood cells (PRBCs) to a patient who had blood loss during surgery. Which action by the new RN requires that you intervene immediately? 1. Waiting 20 minutes after obtaining the PRBCs before starting the infusion 2. Starting an IV line for the transfusion using a 22-gauge catheter 3. Priming the transfusion set using 5% dextrose in lactated Ringer's solution 4. Telling the patient that the PRBCs may cause a serious transfusion reaction

3. Priming the transfusion set using 5% dextrose in lactated Ringer's solution

A transfusion of PRBCs has been infusing for 5 minutes when the patient becomes flushed and tachypneic and says, "I'm having chills. Please get me a blanket." Which action should you take first? 1. Obtain a warm blanket for the patient. 2. Check the patient's oral temperature. 3. Stop the transfusion. 4. Administer oxygen.

3. Stop the transfusion.

A patient is admitted to the intensive care unit with disseminated intravascular coagulation associated with a gram-negative infection. Which assessment information has the most immediate implications for the patient's care? 1. There is no palpable radial or pedal pulse. 2. The patient reports chest pain. 3. The patient's oxygen saturation is 87%. 4. There is mottling of the hands and feet.

3. The patient's oxygen saturation is 87%.

During a home visit to an 88-year-old client who is taking digoxin (Lanoxin) 0.25 mg daily to treat heart failure and atrial fibrillation, you obtain this assessment information. Which finding is most important to communicate to the health care provider? 1. Apical pulse of 68 beats/min and irregularly irregular 2. Digoxin taken with meals 3. Vision that is becoming "fuzzy" 4. Lung crackles that clear after coughing

3. Vision that is becoming "fuzzy"

You receive a change-of-shift report about the following patients. Which one will you assess first? 1. 26-year-old with thalassemia who has a hemoglobin level of 8 g/L and orders for a PRBC transfusion 2. 44-year-old who was admitted 3 days previously in a sickle cell crisis and has orders for a computed tomographic scan 3. 50-year-old with stage IV non-Hodgkin lymphoma who is crying and saying, "I'm not ready to die" 4. 69-year-old with chemotherapy-induced neutropenia who has an oral temperature of 100.1° F (37.8° C)

4. 69-year-old with chemotherapy-induced neutropenia who has an oral temperature of 100.1° F (37.8° C)

You assess a client who has just returned to the recovery area after undergoing coronary arteriography. Which information is of most concern? 1. Blood pressure is 144/78 mm Hg 2. Pedal pulses are palpable at +1 3. Left groin has a 3-cm bruised area 4. Apical pulse is 122 beats/min and regular

4. Apical pulse is 122 beats/min and regular

A resident in a long-term care facility who has venous stasis ulcers is treated with an Unna boot. Which nursing activity included in the resident's care is best for you to delegate to the UAP? 1. Teaching family members the signs of infection 2. Monitoring capillary perfusion once every 8 hours 3. Evaluating foot sensation and movement each shift 4. Assisting the client in cleaning around the Unna boot

4. Assisting the client in cleaning around the Unna boot

You are monitoring a 53-year-old client who is undergoing a treadmill stress test. Which client finding will require the most immediate action? 1. Blood pressure of 152/88 mm Hg 2. Heart rate of 134 beats/min 3. Oxygen saturation of 91% 4. Chest pain level of 3 (on a scale of 10)

4. Chest pain level of 3 (on a scale of 10)

A patient with Hodgkin lymphoma who is receiving radiation therapy to the groin area has skin redness and tenderness in the area being irradiated. Which nursing activity should you delegate to the UAP caring for the patient? 1. Checking the skin for signs of redness or peeling 2. Assisting the patient in choosing appropriate clothing 3. Explaining good skin care to the patient and family 4. Cleaning the skin over the area daily with a mild soap

4. Cleaning the skin over the area daily with a mild soap

4. Based on this information in a client's medical record, which topic will you plan on including in the initial teaching plan for a client who has a new diagnosis of stage 1 hypertension? Health History Physical Exam Social and Diet History • Denies any chronic health problems • Takes no medications currently • Height: 5 feet, 6 inches • Weight: 115 lb (52.3 kg) • BMI: 18.6 • Works as an accountant • 1 glass of wine once or twice weekly • Eats "fast food" frequently 1. Benefits and adverse effects of beta-blockers 2. Adverse effects of alcohol on blood pressure 3. Methods for decreasing dietary caloric intake 4. Low-sodium food choices when eating out

4. Low-sodium food choices when eating out

At 9:00 pm, you admit a 63-year-old with a diagnosis of acute MI. Which finding is most important to communicate to the health care provider who is considering the use of fibrinolytic therapy with tissue plasminogen activator (alteplase [Activase]) for the client? 1. The client was treated with alteplase about 8 months ago. 2. The client takes famotidine (Pepcid) for esophageal reflux. 3. The client has ST-segment elevations on the 12-lead ECG. 4. The client has had continuous chest pain since 8:00 am.

4. The client has had continuous chest pain since 8:00 am.

You are reviewing the complete blood count for a patient who has been admitted for knee arthroscopy. Which value is most important to report to the physician before surgery? 1. Hematocrit of 33% 2. Hemoglobin level of 10.9 g/dL 3. Platelet count of 426,000/mm3 4. White blood cell count of 16,000/mm3

4. White blood cell count of 16,000/mm3

A client comes into the emergency department (ED) with complaints of chest pain. The client is put to bed and oxygen has been started. Diagnostic tests have been ordered; which one will the nurse make sure is completed first? A 12-lead electrocardiogram (ECG) Serum troponin levels A positron emission tomography (PET) scan Arterial blood gases (ABG)

A 12-lead electrocardiogram (ECG)

The nurse is reviewing an assigned group of clients. Which client would require a focused assessment for the presence of peripheral artery disease (PAD)? A 35-year-old with a 10-year history of chronic hypertension A 60-year-old currently in atrial fibrillation A 55-year-old with renal insufficiency A 72-year-old with a 20-year history of type 2 diabetes

A 72-year-old with a 20-year history of type 2 diabetes

The nurse has been assigned the following clients. Which clients are at risk for hypovolemic shock? Select all that apply. A client with rheumatoid arthritis taking gold therapy treatments A client in end-stage kidney failure A client with diabetes insipidus A client in liver failure with esophageal varices A client following a percutaneous extraction of a kidney calculus A client with 60% total body surface burn area

A client in liver failure with esophageal varices A client following a percutaneous extraction of a kidney calculus A client with 60% total body surface burn area

The nurse in a cardiac stepdown unit has received a hand-off shift report for these clients. Which client should the nurse assess first? A client who has just returned from a coronary arteriogram with placement of an intracoronary stent A client who is in heart failure and has gained 2 lb (0.9 kg) in the last 24 hours A client with endocarditis who has a temperature elevation of 101 °F (38.3 °C) and a pulse rate of 100 beats/min A client who was cardioverted from atrial fibrillation 24 hours ago and has four atrial premature beats (contractions)/min

A client who has just returned from a coronary arteriogram with placement of an intracoronary stent

The nurse has received a hand-off report for a group of clients on a telemetry step-down unit. Which client should the nurse assess first? A client with monitor pattern showing a PR interval of 0.24 and occasional premature ventricular beats A client who is in a sinus bradycardia of 58 beats/min and is experiencing orthostatic hypotension A client who was cardioverted from atrial fibrillation an hour ago and is now in sinus rhythm A client who was in second-degree block and had a permanent pacemaker inserted 6 hours ago

A client who was cardioverted from atrial fibrillation an hour ago and is now in sinus rhythm

Which client that requires the most immediate nursing intervention? A client with a diagnosis of anteriolateral myocardial infarction (MI) with frequent multifocal premature ventricular contractions (PVCs) A client receiving a beta-blocker for second-degree heart block who is has a heart rate of 58 beats/min after receiving medication A client who has a newly implanted demand pacemaker with a synchronized rate of 72 that fluctuates between 72 and 76 A client who has experienced episodes of atrial fibrillation and currently has a heart rate of 98 beats/min

A client with a diagnosis of anteriolateral myocardial infarction (MI) with frequent multifocal premature ventricular contractions (PVCs)

What is the cause of sickle cell anemia? Exposure to ionizing radiation A genetically induced production of abnormal hemoglobin S A deficiency of dietary folic acid during pregnancy Long-term use of thiazide diuretics

A genetically induced production of abnormal hemoglobin S

What information should the nurse give to a client prior to a femoral arteriogram? A local anesthetic will be given to lessen discomfort. There are minimal risks associated with the procedure. The radioactive dye that is injected will be removed before the client returns to his unit. A radiopaque substance will be injected into the vein and will be directly tracked.

A local anesthetic will be given to lessen discomfort.

Which client would the nurse assess first after receiving a change-of-shift report? An older adult with chronic myelogenous leukemia (CML) who is refusing the ordered chemotherapy regimen An adult with pernicious anemia who is being discharged and needs teaching about medications and diet A newly admitted young adult client with a history of hemophilia and hemarthrosis of the left knee An adult who has non-Hodgkin lymphoma and is complaining of pain

A newly admitted young adult client with a history of hemophilia and hemarthrosis of the left knee

The nurse is preparing a client for magnetic resonance imaging (MRI). When the nurse reviews the client's history, what information would be most important for the nurse to report to the healthcare provider? The client had a myocardial infarction 2 months ago. The client has an allergy to iodine. A permanent pacemaker was placed 6 weeks ago for third-degree block. Warfarin was started after the last hospitalization.

A permanent pacemaker was placed 6 weeks ago for third-degree block.

Which client is a priority and requires the most rapid assessment and care by the emergency room nurse? A client with a history of polycythemia vera experiencing intermittent claudication and paresthesias of the legs A client with hemophilia who has hemarthrosis of the right knee after falling down A pregnant client with sickle cell anemia who is experiencing excessive fatigue and elevated blood pressure A client with leukemia who has chemotherapy-induced neutropenia, a temperature of 100° F (37.8° C) and has been vomiting for the past 8 hours

A pregnant client with sickle cell anemia who is experiencing excessive fatigue and elevated blood pressure

Which pathophysiologic finding most strongly suggests left-sided heart failure? A decrease in pulse rate An increase in central venous pressure A rise in pulmonary artery diastolic pressure A decline in the mean pulmonary artery pressure

A rise in pulmonary artery diastolic pressure

A client is admitted to the hospital for a diagnostic workup for hypertension. What assessment data gathered by the nurse would confirm this diagnosis? A diastolic pressure fluctuating between 138 mm Hg and 170 mm Hg Decreasing urinary output with a serum potassium level of 5.5 mEq/L (5.5 mmol/L) A pulse pressure greater than 60 beats/min with a 20-point pulse deficit A sustained increase in systolic pressure above 140 mm Hg and diastolic above 90 mm Hg

A sustained increase in systolic pressure above 140 mm Hg and diastolic above 90 mm Hg

After a dental procedure, a client with hemophilia has mild pain. When giving discharge instructions, which analgesic would be prescribed? Morphine Any nonsteroidal anti-inflammatory drug (NSAID) Aspirin Acetaminophen

Acetaminophen

The nurse anticipates that epoetin alfa would be therapeutic for clients with which conditions? Select all that apply. Anemia associated with pregnancy Acquired immune deficiency syndrome (AIDS) taking zidovudine (AZT) Anemia induced by chemotherapy Tuberculosis Systemic lupus erythematous Chronic renal failure

Acquired immune deficiency syndrome (AIDS) taking zidovudine (AZT) Anemia induced by chemotherapy Chronic renal failure

A transtelephonic event monitor is placed on a client with a recent history of syncope. What will be important for the nurse to teach the client about the monitor? Press the recording button when he is exercising. Maintain a log of activities when the monitor is in place. Activate the monitor if he experiences any dizziness or palpitations. Do not remove the monitor leads at any time.

Activate the monitor if he experiences any dizziness or palpitations.

In discharge planning for a client with heart failure, the nurse discusses the importance of adequate rest. What information is most important? A warm, quiet room is necessary Bed rest promotes venous return A hospital bed is necessary Adequate rest decreases cardiac workload

Adequate rest decreases cardiac workload

A client with a diagnosis of acute coronary syndrome is on a cardiac monitor. The nurse interprets the monitor rhythm to be ventricular tachycardia at a rate of 150 beats/min. The client is awake and coherent, and oxygen is being administered at a rate of 6 L/min via a nasal cannula. What is the first nursing action? Immediately defibrillate Administer a lidocaine intravenous (IV) push Initiate transvenous pacing Administer adenosine IV push

Administer a lidocaine intravenous (IV) push

The nurse is preparing to deliver a cardioversion to a client in a supraventricular tachycardia of 150 beats/min. Place the following steps in the order the nurse would implement them. Place the options into the correct order. Place the electrodes on the client's chest. Turn the defibrillator on and set it for synchronized delivery. Administer a sedative. Set joules to 50 and confirm the location of other personnel. Deliver the electrical charge.

Administer a sedative. Place the electrodes on the client's chest. Turn the defibrillator on and set it for synchronized delivery. Set joules to 50 and confirm the location of other personnel. Deliver the electrical charge. First, the nurse would administer the sedative prior to the cardioversion. After the sedative is administered, the nurse would place the electrodes on the client's chest. Next, the nurse turns on the defibrillator and selects the synchronized setting. It is important to select the appropriate level of joules and clear the area of other personnel. The last step would be delivering the electrical charge.

The nurse is preparing to administer iron dextran parenterally. What are important nursing considerations when administering this medication? Select all that apply. Administer it only subcutaneously. Assess the client's vital signs. Administer a test dose before administration. Obtain informed consent. Monitor for signs of anaphylaxis.

Administer a test dose before administration. Monitor for signs of anaphylaxis.

What would be a priority nursing action in caring for the client who has had a synchronized cardioversion? Continue administering the sedative. Assess the back for burns. Administer oxygen. Start on a regular diet.

Administer oxygen.

The nurse is caring for a client on a telemetry unit who has a regular heart rhythm and rate of 60 beats/min; blood pressure 188/98 mm Hg, respiratory rate 18 breaths/min, and temperature 98.8 °F (37 °C). What action will the nurse take? Withhold the clonidine. Administer the digoxin. Administer stat dose of atropine. Withhold the sublingual nitroglycerin.

Administer the digoxin.

A client in the intensive care unit (ICU) has a diagnosis of malignancy-induced disseminated intravascular coagulation (DIC) with mild bleeding. What would be an important initial nursing action? Infusion of fresh frozen plasma Administration of heparin Assess client for thrombosis Administration of intravenous (IV) antineoplastic medication

Administration of intravenous (IV) antineoplastic medication

A licensed practical nurse (LPN) is assisting with the care of a client with neutropenia. Which nursing action would be appropriate for the registered nurse (RN) to delegate to the LPN? Administration of pegfilgrastim injection Assessment of signs and symptoms of infection Teaching about specific dietary changes to reduce infection Development of the discharge plan

Administration of pegfilgrastim injection

The nurse is assessing a client and preparing him for a coronary angiogram. What will be most important for the nurse to document on the health assessment? The client has taken nothing by mouth (NPO) for the last 12 hours. Allergy to shrimp and crab. Pedal and dorsalis pulses +2 bilaterally. Informed consent signed.

Allergy to shrimp and crab.

A client comes into the emergency department (ER) with complaints of midsternal chest pain radiating to the neck and left arm; it is unrelieved by sublingual nitroglycerin. An electrocardiogram (ECG) is obtained. What observation on the ECG or on the cardiac monitor would indicate to the nurse the need to notify the healthcare provider immediately? Atrial tachycardia at a rate of 125; occasional premature beats Premature ventricular beats (PVBs), unifocal in formation at a rate of 4/min An ST segment elevation from the isoelectric baseline A PR interval of 0.20 seconds

An ST segment elevation from the isoelectric baseline The ST segment elevation should be reported immediately. The client may be experiencing acute angina or acute coronary syndrome, and if fibrinolytics are going to be part of the treatment, they need to be started as soon as possible. The goal is to start the fibrinolytics within 30 minutes of the client arriving in the ER. PVBs should be monitored for frequency and change in characteristics, the PR interval is within normal limits, and the atrial tachycardia should be monitored.

Which client would the nurse assess first after receiving a change-of-shift report? A young adult with anemia with a hemoglobin level of 7.6 g/dL (76 mmol/L) who is scheduled to receive a blood transfusion An adult receiving chemotherapy who has a temperature of 102° F (38.9° C) A young adult who was admitted several days ago with sickle cell crisis and is requesting pain medication An older adult with end stage non-Hodgkin lymphoma who is in pain and crying

An adult receiving chemotherapy who has a temperature of 102° F (38.9° C)

The nurse understands that the following is descriptive of most cases of hemophilia. Select all that apply. Women are carriers, and only women are affected with the disease. It is an autosomal-dominant disorder affecting the blood coagulation cascade. It is an autosomal-recessive disorder with a 1 in 4 chance of a child having the disease. An affected male and normal female or female carrier can produce an affected daughter, carrier female, normal male, and affected male. It is an X-linked recessive deficiency of platelets causing prolonged bleeding. It is an X-linked recessive inherited disorder in which a blood clotting factor is deficient.

An affected male and normal female or female carrier can produce an affected daughter, carrier female, normal male, and affected male. It is an X-linked recessive inherited disorder in which a blood clotting factor is deficient.

The nurse is performing an assessment on the lower extremities of a client with peripheral artery disease (PAD). Which finding would prompt the nurse to contact the health care provider immediately? Warm with bounding pulses Edematous lower extremity with slow capillary refill Postural color changes An ankle-brachial index (ABI) of 0.5

An ankle-brachial index (ABI) of 0.5

The nurse is titrating the intravenous (IV) administration of dopamine for treatment of a client's decreased blood pressure. Which response indicates the medication is having the desired effect? An increase in the urinary output A substantial decrease in blood sugar level An increase in pulse with a decrease in pulse deficit An improvement in the quality of breath sounds

An increase in the urinary output

A 70-year-old man is admitted to the hospital for elective repair of an abdominal aortic aneurysm (AAA). The client's history includes hypertension, hyperlipidemia, and smoking. The client asks the nurse what caused the aneurysm. The nurse's response would be based on what information? Aneurysms are caused most often by atherosclerotic plaque formation, which damages the arterial wall. A congenital defect in the arterial wall allows for dilatation of the wall and formation of the aneurysm. Aneurysms are caused by severe obesity and resulting increased intraabdominal pressure. A history of heart disease and renal problems are frequently the cause of the damage to the arterial wall.

Aneurysms are caused most often by atherosclerotic plaque formation, which damages the arterial wall.

The nurse is caring for a patient who is to receive a transfusion of two units of packed red blood cells. After obtaining the first unit from the blood bank, the nurse would ask which health team member in the nurses' station to assist in checking the unit before administration? Unit secretary A physician's assistant Another registered nurse An unlicensed assistive personnel

Another registered nurse

A client is admitted to the telemetry unit. The monitor pattern is atrial fibrillation, and the history reveals the client has been unresponsive to drug therapy for the past several days. What would the nurse anticipate as treatment for this client? Anticoagulant therapy Electrical cardioversion Insertion of a demand pacemaker Intravenous (IV) amiodarone drip

Anticoagulant therapy

An older adult client comes to the clinic with complaints of increased weakness and fatigue. He has a history of heart failure and is currently taking furosemide and digoxin. What would be most important for the nurse to assess on this client? Increased central venous pressure and jugular vein distention Any complaints of gastrointestinal problems and current level of serum potassium Recent weight gain or loss and presence of peripheral edema Bilateral breath sounds and recent history of dyspnea

Any complaints of gastrointestinal problems and current level of serum potassium

A client with hemophilia comes to the emergency department after bumping his knee. The knee is rapidly swelling. What is the first nursing action? Initiate an intravenous (IV) site to begin administration of cryoprecipitate. Perform a type and crossmatch for possible transfusion. Draw blood to determine hemoglobin and hematocrit values. Apply an ice pack and compression dressings to the knee.

Apply an ice pack and compression dressings to the knee.

A client is admitted and has the following laboratory results: white blood cell (WBC) count of 4000/mm3 (4 × 109/L), red blood cell (RBC) count of 4 million/mm3 (4 × 1012/L), and platelets at 135,000/mm3 (135 × 109/L). The client is scheduled for a bone marrow aspiration. What would be the best nursing action in the care of this client after a bone marrow aspiration from the left posterior iliac crest? Recheck blood tests to determine any changes. Administer two tabs of acetaminophen by mouth (po) to control pain. Check vital signs and monitor for changes. Apply pressure to the puncture site for 5 to 10 minutes.

Apply pressure to the puncture site for 5 to 10 minutes.

A client has sclerotherapy for treatment of superficial varicose veins. What would the nurse include in the discharge teaching for this client? Apply the antiembolism stockings every morning before you get out of bed. Take an aspirin every day to prevent the development of clots in your legs. Avoid prolonged sitting; stand whenever you can. Walk rather than run for your exercise.

Apply the antiembolism stockings every morning before you get out of bed.

A client is started on IV unfractionated heparin therapy. What would be appropriate nursing interventions? Select all that apply. Monitor the daily prothrombin time and INR. Begin administration of warfarin subcutaneously on day 3 Assess for signs of bleeding. Monitor the activated partial thromboplastin (aPTT) time. Have the antidote, Vitamin K, available. Anticipate 1000 mg/mL to be added to IV infusion.

Assess for signs of bleeding. Monitor the activated partial thromboplastin (aPTT) time.

The nurse is caring for a client who underwent cardiac catheterization 2 hours ago. What is an important nursing measure? Measure urinary output hourly and maintain continuous cardiac monitoring. Encourage the client to perform slow passive exercise of the affected side to promote circulation. Assess the catheter insertion site and determine distal circulation status. Evaluate apical pulse and determine the presence of a pulse deficit.

Assess the catheter insertion site and determine distal circulation status.

A client is 2 hours post-procedure of having a cardiac catheterization. The nurse is checking the vital signs and monitoring the femoral groin area puncture site. The nurse asks the client a question and notices the client's speech is slurred and the client was slow to respond to the question. What is a priority nursing action? Recheck the puncture area in the groin for bleeding. Assess the client's neurological status. Ask the client if they are having difficulty speaking. Call the cardiac catheterization laboratory and ask what dye was used.

Assess the client's neurological status.

What is the best method for the nurse to do when assessing for pallor in a client with dark skin? Assess for changes in the color in the forearm. Assess the soles of the hands and feet. Assess the conjunctiva and mucous membranes. Assess the back of the client's throat.

Assess the conjunctiva and mucous membranes.

What is an important assessment for the nurse to make before a client goes for a cardiac catheterization? Evaluate the preparation of a preprocedure electrocardiogram (ECG). Determine the client's ability to void from a supine position. Conduct post-procedure teaching regarding bed rest in a supine position. Assess the quality of pulses and the temperature of the client's extremities.

Assess the quality of pulses and the temperature of the client's extremities.

The nurse is caring for a client who is recovering from a myocardial infarction. The nurse is assisting the client to ambulate around the room. The client begins to complain of shortness of breath and "just a little chest pain." What would be the first nursing action? Assist the client back to bed and begin oxygen administration. Check the cardiac monitor and obtain a rhythm sample. Administer morphine subcutaneously immediately. Further assess the characteristics of the chest pain.

Assist the client back to bed and begin oxygen administration.

The nurse obtains a monitor strip on a client admitted to the coronary care unit. The nurse makes the following analysis: P waves are unidentifiable, ventricular rate is 162, R-R interval is grossly irregular, the PR interval not measurable, and the QRS width is 0.20. What would be the nurse's interpretation of this rhythm? Atrial tachycardia Ventricular tachycardia First-degree block Atrial fibrillation

Atrial fibrillation The inability to identify specific P waves or the PR interval, presence of fibrillatory P waves, normal QRS width, fast rate of conduction, and the grossly irregular rhythm are characteristics of atrial fibrillation. There are no P waves, and the QRS is wide with a regular rhythm in ventricular tachycardia. Sinus tachycardia has clearly identifiable P waves. Ventricular fibrillation is grossly irregular with complexes that are of varying shape and duration; no consistent pattern exists. First-degree block is characterized by a PR interval in excess of 0.20; the basic rhythm is sinus and regular.

A client with a diagnosis of acute substernal chest pain is admitted to the unit. What nursing actions can the charge nurse delegate to the licensed practical nurse or licensed vocational nurse (LPN/LVN)? Assess for information about pertinent medical history. Administer 5000 units of heparin intravenous (IV) push. Attach cardiac monitoring leads. Administer 15 mg of morphine IV push.

Attach cardiac monitoring leads.

The nurse is admitting a client who is complaining of chest pain. What would be the first nursing action? Place a pulse oximetry and determine the saturation of peripheral oxygen (SpO2). Administer nitroglycerin sublingually. Obtain the blood pressure and apical pulse rate. Attach the cardiac monitor.

Attach the cardiac monitor.

Which actions should the nurse perform before administering digoxin? Select all that apply. Auscultate the apical pulse. Take the blood pressure. Check the serum digoxin level. Document that the client took the digoxin. Check the laboratory results for hypokalemia. Assess for blurred vision, anorexia, and dysrhythmias.

Auscultate the apical pulse. Check the serum digoxin level. Check the laboratory results for hypokalemia. Assess for blurred vision, anorexia, and dysrhythmias.

An elderly client received one unit of packed red blood cells (PRBCs) in one hour. Which priority nursing assessment should the nurse perform? Take the client's temperature. Auscultate the client's lung sounds. Confirm adequate urinary output. Assess the intravenous catheter site.

Auscultate the client's lung sounds.

The nurse is performing an assessment on a newly admitted client. She notes a pulsating mass in the client's periumbilical area. Which action is a priority? Percuss the periumbilical area. Palpate around the periumbilical area. Auscultate the periumbilical area. Measure the height and length of the pulsating mass.

Auscultate the periumbilical area.

A client has essential hypertension and has been prescribed sustained release (SR) verapamil. What information should the nurse include in teaching the client about verapamil? Select all that apply. Avoid drinking grapefruit juice and eating the fruit. Can crush the pill and take it with applesauce. Increase dietary fiber to prevent constipation. Decrease fluid intake to prevent edema. Expect to have liver enzymes and electrolytes monitored monthly.

Avoid drinking grapefruit juice and eating the fruit. Increase dietary fiber to prevent constipation.

A client with deep vein thrombosis (DVT) being discharged home is to receive low-molecular-weight heparin (LMWH). What would be important information for the nurse to include in the teaching plan? Select all that apply. Avoid massaging the injection site. Schedule a weekly visit to have the partial thromboplastin time measured. Contact the health care provider if the stool appears dark and tarry in color.4 Teach self-injection subcutaneous technique to the client and family. Anticipate the home care nurse to flush the intermittent infusion intravenous (IV) catheter. Explain that vitamin K is the antidote if spontaneous bleeding occurs (e.g., nosebleed).

Avoid massaging the injection site. Contact the health care provider if the stool appears dark and tarry in color.4 Teach self-injection subcutaneous technique to the client and family.

The nurse has an order to transfuse one unit of packed red blood cells to a client with anemia. The client has A+ blood. Which ABO blood types and Rh factors are acceptable for substitution? Select all that apply. A− O+ B− AB+ O− B+

A− O+ O−

A client is 4 hours postoperative following a repair of a thoracic aortic aneurysm. What assessment finding needs to be reported immediately to the surgeon? Abdominal pain BP 194/100 mm Hg Hoarseness Heart rate 58 beats/min

BP 194/100 mm Hg

The nurse has a standing order on the cardiac postoperative unit to notify the healthcare provider if a client's mean arterial pressure (MAP) is less than 70 mm Hg. Which blood pressure (BP) reading would require the nurse to notify the healthcare provider? BP of 112/80 in a client with heart failure BP of 110/60 in a client with a recent postoperative infarction BP of 104/68 in a newly admitted surgical client BP of 108/48 in a 24-hour postoperative client

BP of 108/48 in a 24-hour postoperative client

What activities are appropriate for a client who has had an acute coronary syndrome (myocardial infarction) and has been transferred to a step-down telemetry unit from the cardiac intensive care unit? Bed rest Bathroom privileges and beginning self-care, as tolerated Ambulation in the nursing unit, as desired Beginning cardiac rehabilitation exercises

Bathroom privileges and beginning self-care, as tolerated

Norepinephrine has been ordered for a client in hypovolemic shock. Before administering the drug, which condition should be verified? A heart rate less than 120 beats/min Urine output at least 30 mL/hr Received adequate anticoagulation Been receiving adequate intravenous (IV) fluid replacement

Been receiving adequate intravenous (IV) fluid replacement

A client with coronary artery disease turns on his call light. When the nurse enters the room, he tells the nurse he is experiencing chest pain. What is the first nursing action? Listen to heart sounds for ectopic beats. Auscultate breath sounds and maintain an airway. Determine what the client was doing before onset of pain. Begin oxygen administration at a rate of 4 L/min via a nasal cannula.

Begin oxygen administration at a rate of 4 L/min via a nasal cannula.

A client is admitted to rule out an acute coronary syndrome or myocardial infarction. He has an intravenous (IV) line in place, is attached to a cardiac monitor, and is in a semi-Fowler's position. He suddenly complains of severe substernal chest pain. Which action will the nurse take first? Order stat cardiac enzymes, particularly creatinine phosphokinase-MB (CPK-MB). Determine what medications he received last. Begin oxygen via nasal cannula. Call the doctor and advise him of the status of the client.

Begin oxygen via nasal cannula.

A client begins receiving iron therapy because of iron-deficiency anemia. How should the client administer iron for maximum absorption? With two 8-oz. (240 mL) glasses of water After a glass of skim milk Immediately after a full meal Between meals

Between meals

An older adult client has a 25-year history of chronic pulmonary disease. He is admitted to the hospital with a diagnosis of cor pulmonale or right-sided heart failure. What would the nurse expect to find on the initial assessment of this client? Bilateral pitting edema in the legs A pericardial friction rub Bilateral breath sounds with wheezing and crackles Paroxysmal nocturnal dyspnea

Bilateral pitting edema in the legs

A client who is in the coronary care unit is receiving intravenous nitroglycerin for unstable angina. While the client is receiving the medication, what is an assessment priority? Cardiac rhythm and rate Blood pressure Respiratory rate Urine output

Blood pressure

The nurse is caring for an adult client. The nurse would question the administration of the next dose of nifedipine if which assessment finding is obtained? Blood pressure (BP) is 90/56 mm Hg. Pulse is 68 beats/min apically. K is 3.3 mEq/L (303 mmol/L). Urine output is 200 mL over 4 hours.

Blood pressure (BP) is 90/56 mm Hg.

The nurse has an order to bolus a postoperative client with normal saline if the mean arterial pressure is less than 50 mm Hg. The client's blood pressure is 70/38 mm Hg. What is the nurse's priority action? Continue to monitor the client. Calculate the intake and output. Bolus the client with normal saline. Administer furosemide 80 mg intravenously.

Bolus the client with normal saline.

A client has been asked to keep a diet history for 2 days prior to clinic visit. The client has hypercholesterolemia and hypertriglyceridemia and received nutritional counseling information. The nurse is reviewing the client's diet history. What meal selection indicates that the client is selecting appropriate foods to manage this condition? Hamburger, French fries, diet cola drink Chicken with white sauce over fettuccine noodles, garlic bread Broiled fish, vegetable medley of carrots, broccoli, and onion Pork roast, baked potato with butter and sour cream, fresh peas

Broiled fish, vegetable medley of carrots, broccoli, and onion

The nurse is reviewing recent laboratory reports on an older adult client who has myelogenous leukemia (CML) and noticed a high percentage of blast cells. What does this indicate? CML is in the blast phase, which is more acute and aggressive. There is an increased resistance to infection. CML medication regime is effective. The leukemia is changing from myelogenous to lymphocytic.

CML is in the blast phase, which is more acute and aggressive.

A client with peripheral artery disease is being discharged on clopidogrel. What would be important for the nurse to teach the client about this medication? Select all that apply. When seated, slowly stand to maintain equilibrium. Take the medication in the morning on an empty stomach. Return to the clinic in 2 weeks for evaluation of anticoagulation. Call the doctor if you have episodes of nasal bleeding. Avoid eating grapefruit or drinking grapefruit juice.

Call the doctor if you have episodes of nasal bleeding. Avoid eating grapefruit or drinking grapefruit juice.

Four hours after aortic-femoral bypass graft surgery, the nurse assesses the client and is unable to palpate pulses in the operative leg. The client complains of pain in the leg. What is the first nursing action? Massage the leg and apply warm towels. Elevate the leg and recheck the pulse. Call the healthcare provider immediately. Help the client ambulate.

Call the healthcare provider immediately.

A client is started on rosuvastatin. What would be important for the nurse to discuss with this client? Skin flushing after taking the medication is common. Take the medication in the morning and evening before eating. Call the office should you have muscle aches and pains unrelated to exercise. For the first 2 weeks, get up slowly and make sure you are not dizzy.

Call the office should you have muscle aches and pains unrelated to exercise.

The nurse is reviewing the serum laboratory results for a client who was admitted about 4 hours ago complaining of chest pain and has a tentative diagnosis of possible acute myocardial infarction. What will be the most important laboratory values for the nurse to evaluate? Creatinine kinase (CK) with isoenzyme MB Highly sensitive C-reactive protein (hs CRP) Potassium levels in serum electrolytes Cardiac troponin T and cardiac troponin I

Cardiac troponin T and cardiac troponin I

The nurse understands that which pathophysiology causes a client to develop disseminated intravascular coagulation? Characterized by hemolysis of red blood cells (RBCs) Caused by abnormal and accelerated clotting An autoimmune process affecting the bone marrow and hematopoietic tissue A result of thrombotic processes because of lack of production of thrombin

Caused by abnormal and accelerated clotting

The client returns to his room after a thoracotomy. What would the nurse expect to find if excessive blood loss has occurred during surgery? Central venous pressure (CVP) of 3 cm H2O and urine output of 20 mL/hr Jugular vein distention with the head elevated at 45 degrees Chest tube drainage of 50 mL/hr in the first 2 hours Increased blood pressure (BP) and increased pulse pressure

Central venous pressure (CVP) of 3 cm H2O and urine output of 20 mL/hr

During the night, a client with a diagnosis of acute coronary syndrome (myocardial infarction) is found to be restless and diaphoretic. What is the best nursing action? Check his temperature and determine his serum blood glucose level. Turn the alarms low and promote sleep by decreasing the number of interruptions. Check the monitor to determine his cardiac rhythm and evaluate vital signs. Call the healthcare provider to obtain an order for sedation.

Check the monitor to determine his cardiac rhythm and evaluate vital signs.

A client is admitted with a diagnosis of acute pericarditis. What assessment findings would the nurse expect? Select all that apply. Nausea, vomiting, and anorexia Chest pain that radiates to the left side of the neck and shoulder Extreme fatigue and muscle weakness Chest pain reduced by leaning forward and worsened by lying supine Productive cough with clear sputum

Chest pain that radiates to the left side of the neck and shoulder Chest pain reduced by leaning forward and worsened by lying supine

The nurse would expect which client diagnoses would be prescribed nifedipine? Select all that apply. Chronic stable angina Hypertension Atrial fibrillation Ventricular tachycardia First degree heart block

Chronic stable angina Hypertension

The nurse assesses a patient and documents the following findings: "Skin of the lower leg is leathery, with brawny appearance. 2+ edema noted with eczema. Patient verbalizes skin itches." What condition does the patient likely have? Deep vein thrombosis Chronic venous insufficiency Peripheral arterial disease (PAD) Raynaud's syndrome

Chronic venous insufficiency

A client is being prescribed clopidogrel for treatment of unstable angina. Which medications that the client is taking should be reviewed by the health care provider prior to administering the clopidogrel? Select all that apply. Cimetidine Aspirin Fluconazole Omeprazole Multivitamin Alteplase

Cimetidine Aspirin Fluconazole Omeprazole Alteplase

Which clients will the nurse monitor most closely for development of a febrile transfusion reaction? Select all that apply. Client receiving a rapid infusion of platelets Client receiving multiple transfusions postoperatively Client receiving a postoperative autologous transfusion Client receiving a leukocyte-reduced blood transfusion Client receiving ABO incompatibility whole blood transfusion

Client receiving a rapid infusion of platelets Client receiving multiple transfusions postoperatively

The nurse is reviewing the laboratory results on several clients. Which laboratory result requires follow-up by the nurse? Client with a total cholesterol of 205 mg/dL. Client with an activated partial thromboplastin time (aPTT) of 40 seconds Client with an international normalized ratio (INR) of 5.2. Client with a prothrombin time (PT) of 12 seconds.

Client with an international normalized ratio (INR) of 5.2. When using the INR to monitor warfarin therapy, the desired outcome is usually to maintain the patient's INR between 2.0 and 3.0 regardless of the actual prothrombin time (PT) in seconds. A client with an INR of 5.2 is at risk for bleeding. Although the total cholesterol is slightly elevated (normal value should be less than 200 mg/dL), it is not a priority concern. The other coagulation studies are within normal limits. Normal range of PT is 11 to 12.5 seconds; normal range of aPTT is 30 to 40 seconds; normal range of INR is 0.8 to 1.1.

The nurse is scheduling the plan of care for a group of assigned clients. Which client should be assessed first? Client who is scheduled for discharge following an abdominal aortic aneurysm (AAA) repair Client with blood pressure of 200/128 mm Hg on a nitroprusside drip for a hypertensive crisis Client complaining of discomfort and a wet dressing on the left leg with a venous stasis ulcer Client receiving heparin injections for the past 3 days for a pulmonary embolus

Client with blood pressure of 200/128 mm Hg on a nitroprusside drip for a hypertensive crisis

Which client is a candidate for thrombolytic therapy? Client with acute pericarditis Client with cerebral neoplasm Client with dissecting aortic aneurysm Client with massive pulmonary emboli

Client with massive pulmonary emboli

The nurse is caring for the following clients. Which clients are at risk for secondary hypertension? Select all that apply. Client with an acute urinary tract infection Client with pheochromocytoma Client with primary aldosteronism Client with obesity Client with Cushing disease

Client with pheochromocytoma Client with primary aldosteronism Client with Cushing disease

The charge nurse is making assignments for the unit. Which assignment would be appropriate for the licensed practical nurse or licensed vocational nurse (LPN/LVN)? Client receiving intravenous (IV) nitroprusside for treatment of hypertensive crisis Client with a previous history of pulmonary embolism receiving heparin Client with venous leg ulcers and compression leg dressings Client with postoperative abdominal aortic aneurysm (AAA) repair ready for discharge

Client with venous leg ulcers and compression leg dressings

A client with hypertension arrives for her regular clinic appointment and tells the nurse, "My feet are killing me. These shoes are so tight." What would be important for the nurse to assess first? Client's weight Client's pulse Client's breath sounds Extent of the pedal edema

Client's weight

The nurse is assisting a client who is going to be discharged after an acute myocardial infarction. The client expresses concern regarding his ability to have sexual intercourse. What will the nurse consider as the highest priority when responding? The client should discuss this information with the healthcare provider at the first checkup after discharge. Clients are discouraged from participating in sexual intercourse because of the increased cardiac demands. Clients who can climb two flights of stairs without becoming dyspneic or experiencing angina can have sexual intercourse. Sexual intercourse can most often be resumed within about a month of experiencing the myocardial infarction.

Clients who can climb two flights of stairs without becoming dyspneic or experiencing angina can have sexual intercourse.

A client is admitted to the telemetry unit with a diagnosis of acute gastrointestinal bleed secondary to chronic alcoholism. The nurse assesses the client and documents that the client's tongue is red and shiny. The client stated that she has some numbness in her fingers. The nurse recognizes that the client has signs of which problem? Thalassemia Cobalamin deficiency Folic acid deficiency Increased intake of red meats, eggs, and enriched grain products

Cobalamin deficiency

A client has been taking enalapril for the past 2 weeks. Which finding should be reported to the health care provider? Complaints of a frequent, dry, nonproductive cough Frequent urination, including nocturia Vertigo and dizziness, especially when moving from a sitting to a standing position A change in blood pressure (BP) from 158/100 to 138/90 mm Hg with a drop in the pulse from 90 to 82 beats/min

Complaints of a frequent, dry, nonproductive cough

The nurse is assessing a client who has just returned to the unit after having a percutaneous coronary intervention (PCI). What assessment data would indicate the need for immediate nursing intervention? Blood pressure of 140/88 mm Hg, atrial tachycardia at 104 beats/min Complaints of increasing chest pain Presence of premature ventricular contractions (PVCs) at three per minute A wide QRS complex with a PR interval of 0.24 seconds

Complaints of increasing chest pain The client should not be experiencing an increase in chest pain; the PCI was done to increase the myocardial blood supply. The nurse should start the oxygen and administer nitroglycerin. The tachycardia and the PVCs require continued monitoring for changes. The client is also in first-degree block (PR interval of 0.24) and should be monitored for progression of the block.

A client is admitted to the hospital with the diagnosis of intermittent claudication. With regard to this problem, what would the nurse expect to find on assessment of this client? Petechiae and itching of the lower part of the leg Extensive discoloration and edema of the upper leg Profuse rash and discoloration from trunk down to feet Complaints of pain on walking, relieved by sitting down

Complaints of pain on walking, relieved by sitting down

What would be important to teach a client who is being prescribed furosemide for control of hypertension? Select all that apply. Report any persistent coughing. Confusion can be a side effect. Eat foods low in potassium. Report changes in hearing. Be sure to rise slowly from chairs.

Confusion can be a side effect. Report changes in hearing. Be sure to rise slowly from chairs.

The nurse needs to notify the primary care provider if the client's mean arterial pressure (MAP) is 60 mm Hg or less. The client's blood pressure is 90/50 mm Hg. Which action by the nurse is most appropriate? Continue to monitor the client. Start a running intravenous of normal saline. Immediately notify the primary care provider. Contact the radiology department to order a stat chest x-ray.

Continue to monitor the client.

The nurse is performing a quick estimate of a client's heart rate using the cardiac monitor strip. What is the most rapid and accurate way to determine the heart rate? Print a 30-second strip, count the number of QRS complexes, and multiply by 2. Print a strip, count the number of R waves occurring within a 3-second time marker, and multiply by 20. Count the number of large squares between an R to R interval and divide by 300. Count the number of QRS complexes in a 6-second strip, then multiply by 10.

Count the number of QRS complexes in a 6-second strip, then multiply by 10.

The nurse is reviewing the medication orders for a client who has a history of sickle cell disease. The client is scheduled for an outpatient cyst removal from the arm. What medications would the nurse anticipate for this client? Select all that apply. Hydromorphone Crizanlizumab Endari Methotrexate Hydroxyurea

Crizanlizumab Endari Hydroxyurea

The nurse is preparing discharge teaching for a client with hypertension who is being treated with furosemide and clonidine. The nurse would caution the client about which over-the-counter medications? Antihistamines Acetaminophen Topical corticosteroid cream Decongestant cough preparations

Decongestant cough preparations

After a motor vehicle accident, a client is admitted with a possible lacerated spleen. Which assessment finding would cause the nurse most concern? Elevated temperature Prolonged clotting time Decrease in blood pressure (BP) Complaints of pain

Decrease in blood pressure (BP)

The nurse is caring for a client immediately after repair of an abdominal aortic aneurysm. What changes in vital signs would cause the nurse the most concern? Decrease in blood pressure and increased pulse Decrease in blood pressure and decreased pulse Increase in blood pressure and increased pulse Increase in blood pressure and decreased pulse

Decrease in blood pressure and increased pulse

The nurse is performing an assessment and finds the client has cold, clammy skin, pulse of 130 beats/min and weak, blood pressure of 84/56 mm Hg, and urinary output of 20 mL for the past hour. The nurse would interpret these findings as suggestive of which pathophysiology? Reduction of circulation to the coronary arteries, thus increasing the preload Decreased glomeruli filtration rate resulting in volume overload Stimulation of the sympathetic nervous system causing severe vasoconstriction Decrease in cardiac output and inadequate tissue perfusion

Decrease in cardiac output and inadequate tissue perfusion

A client is in heart failure, and dobutamine has been started via intravenous (IV) continuous infusion. The nurse is evaluating the effectiveness of the medication; what observation would indicate the medication is achieving the desired effects? Decrease in dyspnea with increased urine output Decreased heart rate with an increase in blood pressure Decreased weight and an increase in urine output Blood pressure of 150/80 mm Hg, heart rate of 90 beats/min

Decrease in dyspnea with increased urine output

The nurse is assessing a client for the therapeutic effect of verapamil. What observations would indicate the medication is effective? Decrease in heart rate and blood pressure (BP) Decrease in urine output and decrease in serum glucose levels Decreased BP and increased urine output Increased heart rate and decrease in BP

Decrease in heart rate and blood pressure (BP)

A client with venous thromboembolism (VTE) is started on a continuous infusion of heparin. What is an important nursing action for the client who is on heparin? Monitor rate and quality of peripheral pulses. Notify the healthcare provider if the prothrombin time is greater than 45 seconds. Decrease number of venipunctures by coordinating blood drawn for serum blood studies. Assess the client for adequacy of cardiac perfusion and urinary output.

Decrease number of venipunctures by coordinating blood drawn for serum blood studies.

The nurse is monitoring an intravenous (IV) infusion of sodium nitroprusside. Fifteen minutes after the infusion is started, the client's blood pressure (BP) goes from 190/120 mm Hg to 120/90 mm Hg. What is a priority nursing action? Recheck the BP and call the doctor. Decrease the infusion rate and recheck the BP in 5 minutes. Stop the medication and keep the IV open with D5W. Assess the client's tolerance of the current level of BP.

Decrease the infusion rate and recheck the BP in 5 minutes.

A client has left-sided heart failure and the nurse notes he has symptoms of dyspnea and orthopnea. What is the most immediate short-term goal? Decrease venous return to the heart. Maintain anticoagulation for peripheral edema. Determine the stressors in the client's home life. Maintain renal perfusion to promote urinary output.

Decrease venous return to the heart.

Which laboratory test should the nurse identify as indicative of improvement in a client with rheumatic heart disease? Positive C-reactive protein Hemoglobin, 14.0 g/100 mL (140 g/L) blood White blood cell count, 11,000/mm3 (11 × 109/L) Decreasing erythrocyte sedimentation rate (ESR)

Decreasing erythrocyte sedimentation rate (ESR)

The nurse understands that medications that are given to relieve the pain of stable angina do so by what physiologic action? Increasing heart rate, which increases myocardial blood supply Increasing blood pressure, which increases contractility and myocardial blood supply Increasing sympathetic stimulation to the heart, which causes a decrease in blood pressure Decreasing heart rate and causing vasodilation of coronary arteries

Decreasing heart rate and causing vasodilation of coronary arteries

The nurse is admitting a client who came into the emergency department (ED) with complaints of chest pain. The nurse connects the telemetry monitor and evaluates the cardiac pattern. What would be of most concern? Presence of tachycardia at a rate of 106 beats/min T wave inverted Depression in ST segment Premature ventricular beats at 4 beats/min

Depression in ST segment The nurse should be concerned about any deviation of the ST segment from the isoelectric base line. The ST segment depression may be indicative of myocardial ischemia, whereas ST segment elevation may indicate occlusion. The tachycardia and the inverted T wave are of concern, but the ST segment depression is of more concern. The premature ventricular beats (PVBs) may or may not be a problem; the nurse should monitor the client for any increases in the frequency and characteristics of the PVB pattern.

The nurse is caring for a postoperative client the day of surgery for repair of an abdominal aortic aneurysm. The nurse determines that the client does not have a posterior tibial or dorsalis pedis pulse on his left leg and the leg is cool to touch. What is the best nursing action? Notify the surgeon immediately and anticipate the client will return to surgery. Document the status of the pulses and check them again in an hour. Elevate the legs and cover them with a warm blanket. Determine the status of the client's pulses before surgery.

Determine the status of the client's pulses before surgery.

A client comes into the emergency department (ED) with complaints of severe chest pain and a diagnosis of acute coronary syndrome is made. The nurse has a prescription to begin fibrinolytic treatment with reteplase. What in the client's history would require the nurse to notify the healthcare provider before administering the medication? Diagnosed with stroke 2 months ago Complaints of severe headache before chest pain Colon resection 6 months ago Currently taking digoxin and furosemide

Diagnosed with stroke 2 months ago

The nurse is caring for a client with a history of intravenous drug abuse that was diagnosed with bacterial endocarditis. Which complications will the nurse monitor for? Select all that apply. Difficulty breathing Heart failure Murmur Cerebral vascular accident (CVA) Hypertension Cholelithiasis

Difficulty breathing Heart failure Murmur Cerebral vascular accident (CVA)

The nurse is caring for a client 1 hour postoperative after a femoral-popliteal bypass graft. Which assessment finding should be reported immediately? Doppler pedal pulses present and normal strength; body temperature of 99° F (37.2° C) 2+ peripheral edema and slight pallor at the incision site Diminished pedal pulse and decreased ankle-brachial index measurement Serous drainage from the incision and a small hematoma

Diminished pedal pulse and decreased ankle-brachial index measurement

The nurse observes a client's cardiac rhythm and determines that the client has a heart rate of 56 beats/min and the PR interval is 0.30 on the cardiac monitoring strip. What would be the best nursing action? Document the rhythm and continue to monitor the client. Notify the health care provider of the client's rhythm problem. Administer atropine and document the client's response. Begin oxygen and assess the client's respiratory status.

Document the rhythm and continue to monitor the client.

Which client finding would the LPN/LVN ask the registered nurse to assess further? Radial pulse that is irregular, but strong Pedal pulses which are equal and regular Dorsalis pedis pulse palpable only with Doppler Fingertips that are intermittently red and painful

Dorsalis pedis pulse palpable only with Doppler

The nurse is taking the history of a client with heart failure caused by chronic hypertension. The nurse identifies what data as supportive of the client's medical diagnosis? Dyspnea after walking about half a block Weight loss of 15 lb (6.8 kg) over the last 3 months Lower extremity edema in the evenings Dizziness and fainting when rising too quickly

Dyspnea after walking about half a block

The client has a history of being treated with IV amiodarone for life-threatening ventricular arrhythmias and is being discharged to home with the oral form of the medication. What should be emphasized in the discharge plan that would require immediate notification of the healthcare provider (HCP)? Skin deposits and photosensitivity Dyspnea and cough Malaise, fatigue and ataxia Gastrointestinal disturbances

Dyspnea and cough

The nurse is performing an assessment on a client who is having difficulty controlling his left-sided heart failure. What is associated with this type of heart failure? Systemic venous congestion Dyspnea and pulmonary congestion Increased peripheral edema and anorexia Atrial fibrillation with a heart rate around 110 beats/min

Dyspnea and pulmonary congestion

A client and her husband are positive for the sickle cell trait. The client asks the nurse about the chances of her children having sickle cell disease. The nurse understands that this genetic problem will reflect what pattern in the client's children? One of her children will have sickle cell disease. Only the male children will be affected. Each pregnancy carries a 25% chance of the child being affected. If she has four children, one of them will have the disease.

Each pregnancy carries a 25% chance of the child being affected.

A client is admitted with mitral valve disease and left ventricular dysfunction. What is the most reliable test to determine cardiac status? Electrocardiogram (ECG) Stress test Cardiac angiogram Echocardiogram

Echocardiogram

During the admission assessment of an older adult client who complains of fatigue, shortness of breath, and has a hemoglobin of 8.4 mg/dL (84 mmol/L) and hematocrit of 26%, the nurse notes a slight yellow undertone to the skin. What laboratory test finding would the nurse need to communicate to the healthcare provider immediately? Elevated sedimentation rate Negative guaiac test Elevated serum bilirubin level Slight decrease in serum cobalamin level

Elevated serum bilirubin level

A client has a diagnosis of multiple myeloma. What would be the best nursing action? Encourage a high fluid intake daily. Provide a diet high in calcium. Minimize weight bearing on affected extremities. Maintain bed rest.

Encourage a high fluid intake daily.

The nurse is caring for a client with venous thromboembolism (VTE) in the lower left leg. What is a priority nursing intervention for this client? Maintain both legs in a dependent position to facilitate arterial circulation. Encourage the client to ambulate as tolerated. Avoid compression stockings until the client is actively ambulating. Maintain the client on complete bed rest.

Encourage the client to ambulate as tolerated.

Which medication would the nurse anticipate administering to a client who is at risk for systemic emboli from chronic atrial fibrillation? Sotalol Enoxaparin Atropine Lidocaine

Enoxaparin

The nurse is managing a client with acute disseminated intravascular coagulation (DIC). Which medication order would the nurse question? Cryoprecipitate Low-molecular-weight heparin (enoxaparin) Epsilon aminocaproic acid (EACA) Recombinant human-activated protein C (i.e., drotrecogin alfa)

Epsilon aminocaproic acid (EACA)

The nurse is preparing discharge teaching for a client with aplastic anemia. What will be important to include in the teaching plan? Select all that apply. Take your iron with meals and decrease the amount of green, leafy vegetables in your diet. Establish a balance between rest and activity; avoid excessive fatigue. Rest and supplemental oxygen may be required during periods of dyspnea. Drink a glass of wine in the evening to help increase your appetite. Notify your healthcare provider if you begin to experience frequent bruising. Increase your intake of dairy products (milk and cheese) and protein.

Establish a balance between rest and activity; avoid excessive fatigue. Rest and supplemental oxygen may be required during periods of dyspnea. Notify your healthcare provider if you begin to experience frequent bruising.

During the shift hand-off report, the nurse learns that one of the assigned clients is in first-degree heart block. What is a nursing measure to assess the status of this dysrhythmia? Count the radial pulse for 1 full minute. Determine the cardiac rate at the point of maximum impulse (PMI). Evaluate an electrocardiogram (ECG) or monitor strip. Take hourly pulse checks and correlate with blood pressure.

Evaluate an electrocardiogram (ECG) or monitor strip.

A client becomes increasingly restless and agitated in the afternoon after a morning surgery for repair of an abdominal aortic aneurysm. His vital signs are temperature 100° F (37.8° C), apical pulse increased from 88 to 110 beats/min, respirations increased 18 to 24 breaths/min, and blood pressure (BP) decreased from 120/80 to 116/78 mm Hg. What intervention should the nurse implement first? Administer an as needed (prn) dose of a prescribed analgesic. Evaluate the central venous pressure (CVP) and assess for gastric distention. Begin taking vital signs every hour; increase oxygen to 6 L/min. Evaluate the quality of pedal pulses and assess for pulse deficit.

Evaluate the central venous pressure (CVP) and assess for gastric distention.

During an office visit, a client tells the nurse that he is experiencing pain in the legs after walking, and says the pain stops upon sitting down. What would be the best nursing action? Evaluate the client's legs for presence of edema and discoloration. Evaluate the client's peripheral pulses and compare the quality of pulses on each leg. Assess the client's legs for tortuous veins. Determine if the pain is related to changes in temperature.

Evaluate the client's peripheral pulses and compare the quality of pulses on each leg.

A client has returned following a right-sided cardiac catheterization. What assessment finding is a priority and indicates a complication? Oxygen saturation 96% Expanding groin hematoma Blood pressure 102/66 Pulse rate 100 beats/min

Expanding groin hematoma

A female client is admitted with a diagnosis of myocardial infarction. Although all symptoms can occur in females, what symptoms are atypical for the female client with this diagnosis? Extreme fatigue and dyspnea Pain on inspiration Feeling of heaviness and substernal pressure Diaphoresis with exertion

Extreme fatigue and dyspnea

A client has returned following a left-sided cardiac catheterization. What assessment findings are a priority and indicate complications? Select all that apply. Discomfort at the puncture site Bruising at puncture site Extremity weakness Swallowing difficulty Visual changes

Extremity weakness Swallowing difficulty Visual changes

The nurse is caring for a client with venous leg ulcers on the ankle. What is the correct pathophysiology regarding the exudate in venous ulcers? All inflammatory exudate can be considered infectious. Exudate leads to chronic tissue defects characterized by granulation tissue. Platelets and fibrinogen promote the spread of the exudates. Exudate may be serous, sanguineous, or purulent.

Exudate may be serous, sanguineous, or purulent.

The nurse manager is conducting an in-service education program on types and causes of pacemaker malfunction. When the pacemaker fails to initiate an electrical stimulus when it should fire, this is called what type of pacemaker failure? Failure to pace. Failure to sense. Failure to capture. Failure to catch.

Failure to pace.

A client with atrial fibrillation is transferred from the medical floor to the coronary care unit. The client is started on a direct-acting anticoagulant. What other medication does the nurse anticipate the client will receive? Flecainide Dobutamine Adenosine Atropine

Flecainide

A client develops tachycardia, severe dyspnea, diaphoresis, crackles, and profuse, frothy pink sputum. The nurse would place the client in which position? Sims' Semi-Fowler's Trendelenburg Fowler's position with legs lowered

Fowler's position with legs lowered

What information obtained during a health history interview would relate to the hematologic system? Frequent bruising Constipation Hemoptysis Muscle pain

Frequent bruising

A client has been diagnosed with disseminated intravascular coagulopathy (DIC). The nurse will anticipate administering which of the following fluids? Packed red blood cells (PRBCs) Fresh frozen plasma (FFP) Volume expanders, such as 10% dextrose in water (D10W) Whole blood

Fresh frozen plasma (FFP)

A nurse is providing dietary instructions to a client who has been prescribed diltiazem (Cardizem). Which of the following food items would the nurse instruct the client to avoid? Green leafy vegetables Grapefruit juice Orange juice Red meats

Grapefruit juice

The nurse notes a palpable left inguinal lymph node. Which description would be considered an abnormal finding? Hard, fixed Mobile, firm Nontender and 0.5 cm Firm and 1 cm

Hard, fixed

A client has been prescribed lisinopril for treatment of hypertension. Several days later, the client calls the nurse complaining of a nagging, dry cough and some swelling around the lips and tongue. What would be the best response by the nurse? Have the client withhold the next dose of medication. Tell the client to try a throat lozenge. Ask the client if he has any allergies. Have the client see the health care provider immediately.

Have the client see the health care provider immediately.

You are ambulating a cardiac surgery client who has a telemetry cardiac monitor when another staff member tells you that the client has developed supraventricular tachycardia at a rate of 146 beats/min. In which order will you take the following actions? 1. Call the client's physician. 2. Have the client sit down. 3. Check the client's blood pressure. 4. Administer PRN oxygen by nasal cannula.

Have the client sit down. Administer PRN oxygen by nasal cannula. Check the client's blood pressure. Call the client's physician.

The nurse is assessing a client who has been diagnosed with polycythemia vera. What manifestations will the nurse anticipate finding? Increased fatigue and bleeding tendencies Hemoglobin below 13 mg/dL (130 mmol/L) Headaches, dyspnea, claudication Back pain, ecchymosis, and joint tenderness

Headaches, dyspnea, claudication

The nurse is teaching a client about home care and treatment of venous stasis ulcers on the leg. What should be included in the nurse's instructions? Select all that apply. Dressings do not need to be changed frequently because there is minimal drainage. Healing will be facilitated by wearing leg compression devices. When the client is in the sitting position, he or she should keep the legs elevated. Avoid standing for prolonged periods of time. Cool packs can be applied to the ulcers to decrease inflammation. Soak the affected extremity in warm water every evening.

Healing will be facilitated by wearing leg compression devices. When the client is in the sitting position, he or she should keep the legs elevated. Avoid standing for prolonged periods of time.

A client is ambulating in the hall after an acute myocardial infarction 4 days ago. The nurse is evaluating the client's response to the activity. What assessment data would indicate the client is not tolerating the activity? Respiratory rate increases from 16 to 24 breaths/min. Premature atrial contractions increase from 4/min to 8/min. Heart rate increases from 80 to 108 beats/min. Client complains of being unsteady and weak.

Heart rate increases from 80 to 108 beats/min.

A client with cardiac disease is admitted to the hospital. From the following description, what client data most clearly indicate atrial tachycardia? Heart rate of 96 beats/min, P waves present with each QRS complex, T wave after every other beat P waves present with every other QRS complex, heart rate of 120 beats/min, irregular rhythm Heart rate of 110 beats/min, P waves present before each QRS complex, regular rhythm P waves for every third QRS complex, normal PR interval, heart rate of 90 beats/min

Heart rate of 110 beats/min, P waves present before each QRS complex, regular rhythm

A client is taking nifedipine for chronic stable angina. What will the nurse teach the client about the purpose of the medication? Prevent clots from developing in the coronary arteries. Assist to prevent the development of sinus bradycardia. Dilate the coronary arteries and increase the blood supply. Help slow the heart and decrease the heart's need for oxygen.

Help slow the heart and decrease the heart's need for oxygen.

Which nursing action would be most effective in preventing venous stasis in the postoperative surgical client? Raise the foot of the bed for 1 hour; then lower it to stimulate blood flow. Massage the lower extremities every 6 hours. Facilitate active range of motion of the upper body to stimulate cardiac output. Help the client walk as soon as permitted and as often as possible.

Help the client walk as soon as permitted and as often as possible.

An older adult client has a history of iron-deficiency anemia and has not been consistent with taking iron supplements because of problems with constipation. What would the nurse expect the client's laboratory findings to include? Normal reticulocyte count Red blood cell (RBC) count 5,000,000 mm3 (5 × 1012/L) Hematocrit (Hct) 36% Hemoglobin (Hb) 9.8 g/dL (98 mmol/L)

Hemoglobin (Hb) 9.8 g/dL (98 mmol/L)

The nurse is caring for a client who received spinal anesthesia for a right total hip replacement 1 hour ago. Which finding would cause the nurse to notify the surgeon? Unable to move bilateral lower extremities No sensation noted from the waist down Pedal pulses in right foot weaker than left foot Hemovac drainage of 700 mL bloody drainage

Hemovac drainage of 700 mL bloody drainage

A client has been on heparin for the past 3 days and is now being started on warfarin. The client questions why the healthcare provider is changing the medication. How will the nurse explain this to the client? Warfarin will provide additional prophylaxis against the development of emboli. Heparin has an immediate action and a short half-life; warfarin takes several days to be effective. Both anticoagulants have a synergistic effect in preventing the development of a thrombus and emboli. It is necessary to administer the medications together to maintain long-term coagulation.

Heparin has an immediate action and a short half-life; warfarin takes several days to be effective.

A client has a family history of coronary artery disease (CAD) and atherosclerosis. Which laboratory finding would be of concern to the nurse? Blood urea nitrogen (BUN) of 18 mg/dL Low density lipoprotein (LDL) level of 80 mg/dL High density lipoprotein (HDL) level of 25 mg/dL Glycosylated hemoglobin (Hb A1C) level of 4.2%

High density lipoprotein (HDL) level of 25 mg/dL

An older client is admitted with a diagnosis of left-sided heart failure. In the nursing assessment, the nurse would identify what common condition frequently associated with left-sided heart failure (HF)? History of peripheral vascular disease History of untreated hypertension Current treatment for ventricular dysrhythmias History of chronic obstructive pulmonary disease

History of untreated hypertension

A client with acute coronary syndrome (ACS) has been prescribed the following medications: metoprolol, clopidogrel, and acetaminophen. The client's vital signs are as follows: blood pressure 118/88, heart rate 48 beats/min, respiratory rate 20 breaths/min, and temperature 99.2 °F (37.3 °C). What would be an appropriate nursing action? Notify the healthcare provider of the abnormal vital signs. Hold the metoprolol. Administer only the acetaminophen. Administer all the medications as ordered.

Hold the metoprolol.

A client is in a first-degree block with a PR interval of 0.24. Over the next hour, the nurse observes a lengthening PR interval until the rhythm becomes irregular with QRSs present and no corresponding P wave. What would be the best nursing action? Prepare the client for an emergency insertion of a temporary pacemaker. Document the rhythm, assess the vital signs, and continue to monitor the rhythm. Hold the next dose of propranolol. Administer the as needed (prn) dose of adenosine.

Hold the next dose of propranolol.

A client in sickle cell crisis is admitted to the emergency department (ED). What are the priorities of care in order of importance? Nutrition, hydration, electrolyte balance Hydration, pain management, electrolyte balance Hydration, oxygenation, pain management Hydration, oxygenation, electrolyte balance

Hydration, oxygenation, pain management

A client with the diagnosis of multiple myeloma has extreme bone pain, and recently the family reports to the healthcare provider the client is experiencing increased lethargy and confusion. The nurse would correlate these symptoms with which imbalance? Hypokalemia Hypercalcemia Hyperuricemia Hypoalbuminemia

Hypercalcemia

The nurse is admitting a client with a diagnosis of polycythemia vera. What abnormal laboratory findings would the nurse anticipate? Select all that apply. Hypokalemia Hypernatremia Hyperkalemia Hyperuricemia Thrombocytopenia

Hyperkalemia Hyperuricemia

The nurse is called to an older adult client's room, because the blood transfusion has finished an hour ahead of schedule. The nurse is monitoring the client for transfusion-associated circulatory overload (TACO). Which signs and symptoms would the older client exhibit if they begin to experience TACO? Select all that apply. Hypertension Fever Bounding pulse Dyspnea Hypotension

Hypertension Bounding pulse

The nurse understands that clients who are taking hydrochlorothiazide may experience which problem? Hypoglycemia Hypernatremia Hypokalemia Hypouricemia

Hypokalemia

A client is diagnosed with acute leukemia and received a hematopoietic stem cell transplant 6 weeks ago from a matched donor sibling. The client's total white blood cell count is 5000 mm3 with 55% neutrophils. What are appropriate nursing interventions? Assess the client for signs of infection. Notify the stem cell transplant team of the abnormal results. Immediately call the healthcare provider. Document the laboratory results. Obtain serum electrolytes and urine specimen.

Immediately call the healthcare provider.

A client is admitted to the telemetry unit with an episode of substernal chest pain. His diagnostic studies reveal an elevated ST segment on the electrocardiogram (ECG) and an elevated cardiac troponin level. What would be an appropriate initial nursing goal? Reduce anxiety and relieve pain. Start client on a low-sodium diet and decrease fluid intake. Eliminate family stressors and provide a calm environment. Improve myocardial oxygenation and reduce cardiac workload.

Improve myocardial oxygenation and reduce cardiac workload.

A client is ice skating and is found unresponsive. An automated external defibrillator (AED) was used by the ice skating rink personnel and delivered a shock to the client. Emergency medical services arrived and transferred the client who is breathing, has a heart rate, and is unconscious to the hospital. According to hospital protocol, the client was placed in targeted temperature management. What is the rationale or reason for using targeted temperature management after a sudden cardiac arrest? Increases metabolic needs and fluid volume needs Promotes increased urinary output Improves neurologic outcomes and reduces brain damage Reduces risk of septicemia

Improves neurologic outcomes and reduces brain damage

Which clinical finding would be a cause for concern of a potential cardiovascular problem for a client who is taking epoetin alfa? White blood cell (WBC) count of 12,000/mm3 (12 × 109/L) Increase of hematocrit from 32% to 34% Increase in hemoglobin of 2 g/dL (20 mmol/L) in the past 10 days Platelets of 350,000/mm3 (350 × 109/L)

Increase in hemoglobin of 2 g/dL (20 mmol/L) in the past 10 days

The nurse understands the pathophysiology of a decreased cardiac output seen in heart failure is related which parameter? Increase in myocardial contractility Increase in preload Decrease in afterload Decrease in heart rate

Increase in preload

A client is scheduled for a computerized tomography (CT) scan with contrast to evaluate a mediastinal mass. After the scan, which instruction will the nurse give the client? Increase oral fluids. Avoid eating shellfish for 48 hours. Expect to feel warm and flushed. Have a blood urea nitrogen (BUN) and serum creatinine test performed.

Increase oral fluids.

The client with lymphoma requires two units of fresh frozen plasma to be infused. What is the purpose of the transfusion? Provide nutritional supplement. Enhance oxygenation to tissue. Promote fibrin formation. Increase plasma clotting factors.

Increase plasma clotting factors.

A client with heart failure (HF) has digoxin (digitalis) ordered. In evaluating the therapeutic effectiveness of the drug, what would the nurse expect to find? Increased cardiac output and decreased heart rate Increased cardiac workload and urinary output Diaphoresis with decreased output Increased heart rate with increased respirations

Increased cardiac output and decreased heart rate

Which laboratory report finding would most strongly support a diagnosis of acute myelogenous leukemia? Increased lymphocytes Increased thrombocytes Increased reticulocytes Increased leukocytes

Increased leukocytes

A client is being treated with dopamine and dobutamine for septic shock. What is the desired therapeutic effect of these two drugs? Increased myocardial contractility and cardiac output without increased oxygen demand Decreased renal perfusion and peripheral vasodilatation Decreased cardiac output, vasodilatation, and increased blood pressure Increased renal perfusion, increased blood pressure, and decreased myocardial contractility

Increased myocardial contractility and cardiac output without increased oxygen demand

The nurse is providing discharge instructions to a client who has had a splenectomy. Which hematological abnormality is expected after splenectomy? Decreased leukocytes Increased platelets Decreased hemoglobin Increased eosinophils

Increased platelets

What would the nurse identify as a primary physiologic compensatory mechanism that occurs to maintain cardiac output in heart failure? Decreased catecholamine release Decreased production of antidiuretic hormone Increased production of aldosterone Increased secretion of surfactant

Increased production of aldosterone

A client has been diagnosed with multiple myeloma. What laboratory and diagnostic findings would the nurse expect with this client? Select all that apply. Increased serum creatinine Decreased BUN Bence-Jones protein in urine Hypercalcemia Hyperkalemia Decreased hemoglobin

Increased serum creatinine Bence-Jones protein in urine Hypercalcemia Decreased hemoglobin

Which findings suggest that a client with peripheral artery disease needs urgent attention? Select all that apply. Intermittent claudication Increased tingling in a limb with pain Reduced hair on limbs Sudden numbness or burning in a leg or foot Rest pain that awakens the client at night Dependent rubor when foot is dependent

Increased tingling in a limb with pain Sudden numbness or burning in a leg or foot Rest pain that awakens the client at night

After the administration of mannitol, the nurse should expect which outcome to occur? Decreased extracellular fluid volume Increased tubular reabsorption of water Decreased filtration of nitrogenous wastes Increased tubular excretion of water

Increased tubular excretion of water

A client has heart failure. What is the expected physiologic response of the body when edema is reduced? Increased urinary output Increased respiratory rate Increased real body weight Increased rales across all lung fields

Increased urinary output

An older adult client has been taking chlorothiazide for several months for treatment of blood pressure. What nursing assessment findings would correlate with an electrolyte imbalance? Elevated blood pressure Increased weakness and fatigue Numbness and tingling of extremities Hyperactive bowel sounds

Increased weakness and fatigue

The nurse is administering nitroglycerin intravenously to relieve chest pain. What is the therapeutic action of this medication? Increases glomerular filtration, which results in decreased venous return Enhances contractility of the myocardium, which increases oxygen delivery Produces an immediate analgesic effect and relieves chest pain Increases the coronary blood supply and decreases the afterload

Increases the coronary blood supply and decreases the afterload

The nurse is administering digoxin, 0.25 mg, to a client. The client asks the nurse about the drug's effect on his heart. Which description is correct? Dilates the coronary arteries to supply more blood to the heart Increases the heart rate and stimulates the conduction system to strengthen the heartbeat Increases the force of the contraction of the heart muscle and increases the cardiac output Decreases cardiac workload by increasing dilatation of left ventricle

Increases the force of the contraction of the heart muscle and increases the cardiac output

The cardiac monitor indicates that a client is in normal sinus rhythm with frequent premature ventricular contractions (PVCs). If the condition is not resolved, what would the nurse expect next? Atrial flutter Impending heart block Ventricular tachycardia Increasing cardiac irritability

Increasing cardiac irritability

An older adult client is admitted in heart failure. What observation by the nurse indicates the client's condition is getting worse? Increasing irritability and confusion Blood pressure 160/98 mm Hg, pulse 110 beats/min Arterial blood gases showing a significant decrease in the pH and partial pressure of carbon dioxide (PCO2) Urinary output of 60 mL/hr and crackles heard at the base bilaterally

Increasing irritability and confusion

A nurse is providing discharge teaching to a client after a splenectomy. The nurse would include teaching about the increased risk of which of the following? Select all that apply. Infection Bleeding tendencies Electrolyte imbalance Immunologic deficiencies Anemia Chronic abdominal pain

Infection Immunologic deficiencies

A client with a diagnosis of disseminated intravascular coagulation (DIC) has the following assessment findings: blood pressure 78/56 mm Hg, temperature 102.6° F (39.2° C), respirations of 24 breaths/min, and complaints of severe neck and back pain. Which nursing action should the nurse implement first? Infuse 500 mL of 0.9% saline over 30 minutes. Administer acetaminophen. Draw stat coagulation study blood work. Give morphine sulfate intravenously (IV).

Infuse 500 mL of 0.9% saline over 30 minutes.

The nurse is administering fresh frozen platelets (FFP) to a client who has been diagnosed with disseminated vascular coagulation (DIC). What are the nursing implications in administering the FFP? Use straight intravenous (IV) tubing with 5% dextrose in water (D5W) as the primary infusion. Warm the FFP for 15 minutes in warm water prior to infusion. Initiate a separate IV site with a 22-gauge needle. Infuse FFP as rapidly as the client will tolerate, usually over 30 to 60 minutes.

Infuse FFP as rapidly as the client will tolerate, usually over 30 to 60 minutes.

The nurse is caring for a client on the operative day after repair of an abdominal aortic aneurysm. The client's central venous pressure (CVP) has decreased from 10 to 8 cm H2O, urine output has been 20 mL/hr for 2 hours, and the blood pressure (BP) has gradually decreased from 128/88 mm Hg to 100/70 mm Hg. The nurse notifies the health care provider and would anticipate what order? Increase intravenous (IV) rate to 150 mL/hr and prepare the client for emergency surgery. Infuse a bolus of 250 mL of lactated Ringer's solution and obtain a serum blood urea nitrogen (BUN) and creatinine. Obtain a stat hemoglobin and hematocrit and infuse two units of whole blood. Obtain a stat chest x-ray and increase oxygen flow from 4 L/min to 6 L/min.

Infuse a bolus of 250 mL of lactated Ringer's solution and obtain a serum blood urea nitrogen (BUN) and creatinine.

The nurse is preparing blood transfusions for an adult, who is 84-years-old. What are important considerations in administering the blood transfusions that are ordered? Monitor urine output hourly. Infuse blood over 2 to 4 hours. Take vital signs every 30 minutes. Administer second transfusion immediately following the first transfusion.

Infuse blood over 2 to 4 hours.

A client has an order for one unit of packed cells. What is a correct nursing action? Initiate the intravenous (IV) with 5% dextrose in water (D5W) to maintain a patent access site. Initiate the transfusion within 30 minutes of receiving the blood. Monitor the client's vital signs for the first 5 minutes. Monitor the client's vital signs every 2 hours during the transfusion.

Initiate the transfusion within 30 minutes of receiving the blood.

The nurse is explaining to the family of a client about the characteristics of Hodgkin disease. Which characteristics are common findings? Select all that apply. Is a benign lymphoma that is outside of the lymph nodes Insidious onset of symptoms Pain at affected lymph node sites caused by ingestion of small amounts of alcohol Absence of hepatomegaly and splenomegaly May be caused by Epstein-Barr virus or exposure to occupational toxins Painless enlargement of cervical, axillary, inguinal, or mediastinal lymph nodes

Insidious onset of symptoms Pain at affected lymph node sites caused by ingestion of small amounts of alcohol May be caused by Epstein-Barr virus or exposure to occupational toxins Painless enlargement of cervical, axillary, inguinal, or mediastinal lymph nodes

Which of the following characteristics would be more indicative of an arterial leg ulcer rather than a venous leg ulcer? Select all that apply. Intermittent claudication Thick, hardened, and indurated skin Frequent pruritus Capillary refill greater than 3 seconds Irregularly shaped

Intermittent claudication Capillary refill greater than 3 seconds

What would the nurse anticipate on the assessment of a client who has developed an acute peripheral arterial occlusion of the left lower leg? Select all that apply. Intermittent claudication Bounding left pedal pulse Cool skin temperature on the left leg Elevated blood pressure Paresthesia in the left leg Mottled extremity with thickened toenails

Intermittent claudication Cool skin temperature on the left leg Paresthesia in the left leg Mottled extremity with thickened toenails

A patient with a venous thromboembolism (VTE) and co-morbidities of coronary artery disease and diabetes mellitus is being admitted to the medical unit. Which medication does the nurse anticipate will be prescribed for this​ client? IV agratoban (Acova) Intravenous unfractionated heparin Subcutaneous unfractionated heparin Subcutaneous low-molecular-weight heparin

Intravenous unfractionated heparin

A nurse is assessing a client diagnosed with anemia related to gastrointestinal (GI) bleeding. Which prescription would the nurse expect at discharge? Iron B vitamins High-fiber diet Enoxaparin injections

Iron

The nurse anticipates that a client who has a deficiency in which of the following minerals is likely to be at risk for developing anemia? Iron Calcium Zinc Magnesium

Iron

What is the desired action of dopamine when administered in the treatment of shock for a client experiencing acute renal failure? It increases myocardial contractility. It rapidly leads to bradycardia. It causes rapid vasodilation of the vascular bed. It supports renal perfusion by dilation of the renal arteries.

It supports renal perfusion by dilation of the renal arteries.

The nurse is caring for a client with cor pulmonale. What nursing assessment information correlates with an increase in venous pressure? Decreased pulsations at the point of maximum impulse Bradycardia, restlessness, and an increase in respiratory rate Jugular vein distention with the client sitting or with a 45-degree head elevation Crackling sounds over the lower lobes with the client in an upright position

Jugular vein distention with the client sitting or with a 45-degree head elevation

The nurse is caring for a client with right-sided heart failure. What nursing assessment information correlates with an increase in venous pressure? Jugular vein distention with the client sitting or with head at a 45-degree angle Crackling sounds over the lower lobes with the client in an upright position Bradycardia, restlessness, and an increase in respiratory rate Decreased pulsations at the point of maximum impulse

Jugular vein distention with the client sitting or with head at a 45-degree angle

A client with cardiac disease has serum laboratory diagnostics done, and the results are as follows: potassium, 4.5 mEq/L (4.5 mmol/L); blood glucose, 130 mg/dL (7.2 mmol/L); elevated levels of cardiac troponin; blood urea nitrogen (BUN), 42 mg/dL (4.2 mmol/L); and urine specific gravity, 1.001. Which nursing intervention is most appropriate? Keep the client in bed; assess for chest pain and dysrhythmias. Evaluate the total urinary output and correlate with intake over last 24 hours. Increase the client's intake of fluids because the client is dehydrated. Perform another blood glucose determination to evaluate the trend of blood sugar.

Keep the client in bed; assess for chest pain and dysrhythmias.

The nurse is caring for a client with hemophilia who is experiencing bleeding into the knee joints. What is the best nursing care for this client regarding joint mobility and activity? Encourage short walks around the room every 2 hours. Keep the joint immobilized and maintain bed rest for the client. Gently put the legs through passive range of motion every 4 hours. Keep the legs wrapped with elastic bandages and immobilized in splints.

Keep the joint immobilized and maintain bed rest for the client.

A client is being discharged with a prescription for sublingual nitroglycerin (NTG) tablets. What is important to teach the client about this medication? Take the nitroglycerin with a full glass of water and lie down. Keep the nitroglycerin in a dark, tightly closed container. Take two nitroglycerin tablets together for severe chest pain. Take a nitroglycerin tablet if chest pain persists for 10 minutes.

Keep the nitroglycerin in a dark, tightly closed container.

A woman is newly diagnosed with Raynaud's disease. What would be important for the nurse to teach this client? Keep your hands warm; wear gloves when handling cold objects. Maintain regular exercise of about 30 minutes each day. Return to the clinic for blood pressure checks. Avoid prolonged standing.

Keep your hands warm; wear gloves when handling cold objects.

The nurse is explaining to a client the results of recent laboratory blood work. Which laboratory results would be a risk factor for atherosclerosis? Triglycerides of 125 mg/dL (1.4 mmol/L) HDL level of 75 mg/dL (1.9 mmol/L) LDL level of 150 mg/dL (3.9 mmol/L) Total cholesterol of 188 mg/dL (4.9 mmol/L)

LDL level of 150 mg/dL (3.9 mmol/L)

The nurse is preparing to administer medications to a group of clients. Which medications have a first dose effect? Select all that apply. Lovastatin Lisinopril Ticlopidine Prazosin Hydrochlorothiazide

Lisinopril Prazosin

Which instruction should be included in discharge teaching for the client with a new prescription for simvastatin? Flushing occurs in almost all individuals. Sedation is common, but will decrease with time. Liver enzyme levels should be monitored every few months. Watch closely for occurrence of postural hypotension.

Liver enzyme levels should be monitored every few months.

A client recovering from an acute episode of thrombophlebitis is being treated with warfarin. In planning discharge teaching, it would be important for the nurse to include what information? Maintain a constant intake of cauliflower, mayonnaise, and green leafy vegetables. Limit dairy products because they tend to decrease the medication's effectiveness. Do not walk for exercise; aerobic exercises are more effective. Maintain a daily record of intake and output to evaluate renal function.

Maintain a constant intake of cauliflower, mayonnaise, and green leafy vegetables.

The client is being evaluated for cardiac dysrhythmias with a Holter monitor. What would be important for the nurse to teach the client about the monitor? The electrodes should be removed when you take a shower or bath. Maintain an activity log while the monitor is in place. Short, sharp twinges may be felt when the monitor is recording. Call the office each day to transmit the recordings.

Maintain an activity log while the monitor is in place.

A client with a history of an abdominal aortic aneurysm is admitted for surgical repair. During the nursing assessment on admission the client begins to complain of severe back pain. The client's vital signs are blood pressure (BP) 100/60 mm Hg, pulse 130 beats/min, and respirations 30 breaths/min. The nurse notifies the surgeon regarding the client's status. What would the nurse anticipate the treatment to include? Maintain bed rest, keep the client nothing by mouth (NPO) and prepare for emergency surgery. Notify radiology for a stat x-ray of the abdomen and closely monitor vital signs. Prepare for a paracentesis to determine the presence of blood in the abdomen. Prepare for insertion of a pulmonary artery catheter to monitor cardiac status.

Maintain bed rest, keep the client nothing by mouth (NPO) and prepare for emergency surgery.

A patient is diagnosed with a small 4 cm abdominal aortic aneurysm (AAA). What is the best nonsurgical intervention to decrease the risk of rupture of an aneurysm and to slow the rate of enlargement? Maintenance of normal blood pressure and avoidance of hypertension Bedrest until there is shrinkage of the aneurysm Heparin and Coumadin therapy to decrease clotting Intra-arterial thrombolytic therapy

Maintenance of normal blood pressure and avoidance of hypertension

A client is being seen in the clinic for problems of venous stasis and venous insufficiency. Compression or antiembolism stockings have been ordered. What information would be important for the nurse to teach the client regarding the stockings? Put the stockings on each night when you go to bed and remove them in the morning. Make sure the hole at the end of the stockings is under the toes. If the hose are not long enough to go to your thigh, then roll them down. Wear the stockings for extended periods when you are sitting and your feet swell.

Make sure the hole at the end of the stockings is under the toes.

A client is in atrial fibrillation; all pharmacologic attempts to convert the client to sinus rhythm have not been successful. The client is scheduled for an elective cardioversion. What action will the nurse take for this procedure? Administer an analgesic as soon as the cardioversion is completed. Make sure the synchronizer is in the "on" position before cardioversion. Set the level at 250 joules for cardioversion of atrial fibrillation. Make sure the incision is covered before administering the cardioversion.

Make sure the synchronizer is in the "on" position before cardioversion.

A patient is being evaluated for thrombolytic therapy. What are absolute contraindications for this procedure? (Select all that apply.) Malignant intracranial neoplasm Intracranial hemorrhage Major trauma in the last 12 months Suspected aortic aneurysm Pregnancy Ischemic stroke within 3 months

Malignant intracranial neoplasm Intracranial hemorrhage Ischemic stroke within 3 months

The nurse is planning home care for a client with venous leg ulcers. What would be most important for the nurse to include in her teaching? Apply the antibiotic cream twice a day. Apply a hydrocolloid dressing over the open ulcers. Massage both legs whenever they are in the dependent position. Return for regular checks of clotting levels.

Massage both legs whenever they are in the dependent position.

The nurse would identify which criteria as critical to the effectiveness of fibrinolytic therapy? Continuous intravenous (IV) heparin infusion is started, followed by initiation of the fibrinolytic therapy. Complete coagulation studies are obtained; prothrombin time must be within normal range. Medication is administered within 30 minutes of the client being diagnosed with coronary artery syndrome. A complete history and physical are performed to rule out potential problem with renal and liver function.

Medication is administered within 30 minutes of the client being diagnosed with coronary artery syndrome.

The nurse is caring for a patient who complains of angina, dizziness, and dyspnea. Which medication would the nurse anticipate the health care provider will prescribe? Metoprolol Atropine Lidocaine Warfarin

Metoprolol

What would be the best nursing action in the care of a client with an acute exacerbation of polycythemia vera? Maintain bed rest. Monitor pulse oximetry. Administer oral iron supplements. Monitor fluid hydration status.

Monitor fluid hydration status.

A client is receiving an infusion of tissue plasminogen activator (tPA) for treatment of an acute myocardial infarction. What are important nursing actions? Select all that apply. Monitor for hemorrhage. Insert a urinary catheter. Administer medications subcutaneous or intramuscular. Monitor for signs of pulmonary toxicity. Assess client's orientation to time, place, and person.

Monitor for hemorrhage. Assess client's orientation to time, place, and person.

A client has been diagnosed with pernicious anemia. What will the nurse teach this client regarding the medication he will need to take after he goes home? Monthly vitamin B12 injections will be necessary. Daily ferrous sulfate (in oral form) will be prescribed. Coagulation studies are important to monitor the effect of medications. He should reduce his intake of leafy, green vegetables to decrease vitamin K.

Monthly vitamin B12 injections will be necessary.

The nurse is concerned that a client has early symptoms of digoxin toxicity. Which symptoms support this concern of early digoxin toxicity? Select all that apply. Nausea Anorexia Vomiting Dysrhythmias Yellow tinge to vision

Nausea Anorexia Vomiting Yellow tinge to vision

The nurse understands that the cardiovascular complications of sustained hypertension on target organs include which of the following? Select all that apply. Nephrosclerosis Aortic regurgitation Stroke Left ventricular hypertrophy Deep vein thrombosis Retinal damage

Nephrosclerosis Stroke Left ventricular hypertrophy Retinal damage

The nurse is reviewing the complete blood count (CBC) of a client admitted with an elevated temperature and acute abdominal pain. Which information will be most important for the nurse to communicate to the healthcare provider? Hemoglobin 11.0 g/dL (110 mmol/L) and hematocrit 30% Platelets 148,000/mm3 (148 × 109/L) Neutrophils 87% Potassium 3.4 mEq/L (3.4 mmol/L)

Neutrophils 87%

What medication would the nurse anticipate administering to a client who is experiencing a hypertensive crisis? Dopamine Nitroprusside Dobutamine Vasopressin

Nitroprusside

The nurse is preparing to start an intravenous (IV) infusion before the administration of a unit of packed red blood cells (PRBCs). What fluid will the nurse select to maintain the infusion before hanging the unit of blood? 5% dextrose in water (D5W) D5W/0.45% NaCl Lactated Ringer solution Normal saline (0.9% NaCl)

Normal saline (0.9% NaCl)

A young woman comes to the clinic complaining of dizziness, weakness, and palpitations. What will be important for the nurse to evaluate initially when obtaining the health history? Activity and exercise patterns Nutritional patterns Family health status Coping and stress tolerance

Nutritional patterns

An older adult client comes into the emergency department (ED) stating that he has no appetite, is nauseated, his heart feels funny, and he has noticed a haziness in his vision. The client states that he has been taking an antihypertensive drug and digoxin (digitalis) for more than a year. Based on the presenting symptoms, what would be the priority nursing action? Obtain an order for serum potassium and digitalis levels. Perform a neurologic assessment to determine whether he has one-side weakness. Assess lungs for decreased breath sounds and/or adventitious breath sounds. Obtain an order for an electrocardiogram (ECG) and determine cardiac enzyme levels.

Obtain an order for serum potassium and digitalis levels.

The nurse is analyzing a client's cardiac monitor strip. The nurse would identify what area of the cardiac complex as representing the conduction time of the impulse from the sinus atrial (SA) node through the atrium and into the bundle branches? QT interval PR interval QRS width R to R interval

PR interval The PR interval represents depolarization of the atria, AV node, bundle of His, bundle branches, and progression through the Purkinje fibers to the point of depolarization of the ventricular cells. Atrial depolarization is represented by the P wave, ventricular depolarization is represented by the QRS, and delayed conduction through the ventricles will increase the width of the QRS. Depolarization of the ventricular cells and repolarization of the conduction system are represented by the QT interval. The R to R interval determines the rate.

A client is admitted to the coronary care unit with a heart rate of 160 beats/min and is diagnosed with paroxysmal supraventricular tachycardia (PSVT). What other assessment data would the nurse anticipate? Bradypnea Flushing of the skin Palpitations Elevated blood pressure

Palpitations

The nurse is providing preoperative care for a client who is scheduled for cardiac surgery. During the preoperative preparation, what is an important nursing action? Perform a thorough nursing assessment to provide an accurate baseline for evaluation after surgery. Discuss with the client the steps of myocardial cellular metabolism and the anticipated surgical response. Provide preoperative education regarding the mechanics of the cardiopulmonary bypass machine. Discuss with the client and family the anticipated amount of postoperative chest tube drainage.

Perform a thorough nursing assessment to provide an accurate baseline for evaluation after surgery.

What would be an appropriate nursing intervention for the nurse to delegate to the licensed practical nurse or licensed vocational nurse (LPN/LVN) in the care of a client with a venous ulcer who has an Unna boot? Change the Unna boot dressing daily. Teach the client about how to care for the Unna boot. Perform neurovascular checks on the extremity with the Unna boot every 8 hours. Remove the Unna boot and cleanse the ulcer area daily.

Perform neurovascular checks on the extremity with the Unna boot every 8 hours.

Which assessment finding would most likely indicate that the client is developing right-sided heart failure? Select all that apply. Orthopnea Peripheral edema Increased urine output Pink-tinged sputum Abnormal heart sounds Distended jugular veins

Peripheral edema Distended jugular veins

The nurse is assessing the lower extremities of a client with early chronic venous insufficiency. What would the nurse expect to find? Dependent cyanosis Muscle atrophy Diminished peripheral pulses Pitting edema of the lower extremities

Pitting edema of the lower extremities

The nurse is assessing a client who has just developed an irregular pulse rhythm; on further assessment the nurse determines there is a pulse deficit. What is the best nursing action? Continue hourly blood pressure (BP) checks. Prepare the client for a transesophageal echocardiogram. Assess and compare the peripheral pulse rate with the apical pulse rate. Place the client on a cardiac monitor.

Place the client on a cardiac monitor.

The nurse is administering alteplase (tPA) to a client who has been diagnosed with acute coronary syndrome (ACS). What are important nursing implications for this medication? Monitor the electrocardiogram (ECG) for dysrhythmias. Place the client on bleeding precautions. Monitor urine output hourly. Monitor for activity intolerance.

Place the client on bleeding precautions.

The nurse responds to a client's cardiac monitor alarm; the client is unconscious with no pulse or respirations. The cardiac monitor pattern is irregular with no identifiable QRS complexes. The nurse calls for assistance and for the defibrillator and emergency cart. What is the next nursing action? Place in correct order: Open the airway. Immediately begin cardiac compressions. Defibrillate as soon as the cart and defibrillator arrive. Begin ventilations via bag valve mask with oxygen at 100%. Position the client on his or her back.

Position the client on his or her back. Immediately begin cardiac compressions. Open the airway. Begin ventilations via bag valve mask with oxygen at 100%. Defibrillate as soon as the cart and defibrillator arrive. The client is unconscious, with no respirations or pulse; cardiopulmonary resuscitation (CPR) should begin immediately. The client's cardiac rhythm appears to be ventricular fibrillation; therefore the nurse should anticipate defibrillating the client. However, the nurse should start CPR before the defibrillator arrives; the nurse would continue CPR while someone else is preparing the defibrillator. After calling for help and positioning the client on his or her back, the nurse should start chest compressions immediately ("push hard and push fast") at a rate of 100/min and a depth of 2 inches (5.1 cm) at a ratio of 30 compressions/2 breaths. Next, the nurse would open the airway, start ventilations using a bag valve mask 100% FIO2, and defibrillate as soon as the defibrillator arrives.

A client with a history of heart failure is being treated with enalapril and spironolactone. When reviewing the client's chart, which laboratory data would be the most significant to monitor? Creatinine and blood urea nitrogen (BUN) Glucose Osmolarity Potassium

Potassium

The client who is receiving digoxin and furosemide is at risk for developing digitalis toxicity. Which of the following serum laboratory values would the nurse correlate with the development of digitalis toxicity? Sodium level of 135 mEq/L (135 mmol/L) Blood urea nitrogen (BUN) of 35 mg/dL (12.5 mmol/L) Calcium level of 6 mg/dL (1.50 mmol/L) Potassium level of 3.0 mEq/L (3.0 mmol/L)

Potassium level of 3.0 mEq/L (3.0 mmol/L)

The nurse is preparing to administer spironolactone to a client. After assessing the client, what data indicate the need to withhold the medication? Potassium level: 5.8 mEq/L (5.8 mmol/L) Apical pulse rate: 58 beats/min Blood pressure (BP) of 130/90 mm Hg Urine output: 30 mL/hr

Potassium level: 5.8 mEq/L (5.8 mmol/L)

A client at a concert develops ventricular fibrillation following a drug overdose and collapses. The nurse working with the EMS unit has determined unresponsiveness. List the actions in order of priority that the nurse should take. Establish intravenous access. Insert an oral airway and begin ventilations. Prepare for defibrillation. Initiate cardiopulmonary resuscitation (CPR).

Prepare for defibrillation. Establish intravenous access. Insert an oral airway and begin ventilations. Initiate cardiopulmonary resuscitation (CPR). The priority nursing action is to defibrillate the client before any other resuscitative measures are initiated. After the defibrillation procedure, the nurse should start an intravenous access line, then place an oral airway. Cardiopulmonary resuscitation is initiated after unsuccessful defibrillation.

The nurse is caring for a client who has hypersplenism. What laboratory test finding would indicate that the client has splenomegaly? Presence of Reed-Sternberg cells Elevated red blood cell (RBC) count Increased Bence-Jones protein in urine Presence of Howell-Jolly bodies in a blood smear

Presence of Howell-Jolly bodies in a blood smear

The nurse is participating in the cardiopulmonary resuscitation (CPR) of a client who has had a cardiac arrest. In determining the effectiveness of the chest compressions in the resuscitation, what would be important for the nurse to evaluate? Presence or absence of dilated pupils Presence of a palpable carotid pulse A capillary refill within 2 seconds Skin remains pink and warm to touch

Presence of a palpable carotid pulse

The nurse has been assigned a group of cardiac clients. What would be the most important information for the nurse to check on the initial evaluation of each client? Select all that apply. Presence of cardiac pain Medications taken before hospitalization Presence of jugular vein distention (JVD) Heart sounds and apical rate Presence of diaphoresis History of difficulty breathing

Presence of cardiac pain Presence of jugular vein distention (JVD) Heart sounds and apical rate Presence of diaphoresis

A nurse is preparing to administer an intramuscular (IM) injection of iron. Why should the nurse change the needle after drawing the medication into the syringe? Prevent staining of the skin. Improve absorption of the medication. Decrease the risk of life-threatening anaphylaxis. Administer the medication using the Z-track injection technique.

Prevent staining of the skin.

After cardiac catheterization, the nurse explains to the client the importance of increasing his fluid intake. What is the goal of this nursing intervention? Replace blood loss during the procedure. Provide hydration so that blood clots will not form. Promote excretion of the contrast dye used during coronary angiography. Provide intake because solid food is prohibited for 24 hours after catheterization.

Promote excretion of the contrast dye used during coronary angiography.

Which medication should be readily available while a client is receiving a continuous intravenous (IV) heparin infusion? Procainamide Protamine sulfate Atenolol Calcium gluconate

Protamine sulfate

A client is started on the direct oral anticoagulant (DOAC), dabigatran. What are important nursing considerations when teaching the client about the medication? Prothrombin time and INR are not required for monitoring. There is no antidote for the medication to reverse its effect. May immediately stop taking the medication if bleeding occurs. Is administered parenterally, client will need to return to the clinic weekly for the injection.

Prothrombin time and INR are not required for monitoring.

A client has been taking amiodarone for a year. At the clinic visit, the client starts to exhibit dyspnea and coughing and complains of chest pain. With what problem would the nurse associate these symptoms? Cardiac dysrhythmias Angina pectoris Pulmonary toxicity Heart failure

Pulmonary toxicity

A client is receiving propranolol for treatment of his cardiac dysrhythmias. What observation would indicate to the nurse to hold the next dose of medication? Decrease in blood pressure from 160/90 to 128/70 mm Hg Pulse decrease from 80 to 50 beats/min Monitor rhythm of sinus tachycardia and premature atrial beats Complaints of mild chest pain relieved by nitroglycerin

Pulse decrease from 80 to 50 beats/min

The vital signs of a client with cardiac disease are as follows: blood pressure of 102/76 mm Hg, pulse of 52 beats/min, and respiratory rate of 16 breaths/min. Atropine (atropine sulfate) is administered by intravenous (IV) push. What nursing assessment indicates a therapeutic response to the medication? Pulse rate is 70 beats/min. Systolic blood pressure is 122/80 mm Hg. Pupils are dilated. Oral secretions have decreased.

Pulse rate is 70 beats/min.

A client is admitted for evaluation of his permanent pacemaker. What data would confirm that the pacemaker is not working correctly? Pulse rate of 96 beats/min with regular rate and rhythm Irregular pulse rate with premature ventricular beats (PVCs) Atrial premature beats shown on the monitor Pulse rate of 48 beats/min with premature ventricular beats

Pulse rate of 48 beats/min with premature ventricular beats

Which assessment findings are consistent with a complication related to leukemia? Elevated white blood cell (WBC) count Pyuria and urgency Chest pain and shortness of breath Decreased urinary output and elevated serum potassium

Pyuria and urgency

The nurse is evaluating the cardiac monitor strip for a newly admitted older adult client. Which observation of the 6-second monitor strip would cause the most concern? P wave for each QRS complex Three atrial ectopic beats Four QRS complexes on the strip QRS duration of 0.24 seconds

QRS duration of 0.24 seconds If there are only four QRS complexes on the 6-second strip, the client's heart rate is 40 beats/min. The three atrial ectopic beats will cause an irregular pulse, and the wide QRS may be a bundle branch block; however, both of these are not as significant as the slow heart rate. A P-wave with each QRS is normal.

The nurse is assessing an older adult client with a history of heart failure. The nurse notes that the client has jugular vein distention (JVD) when the client is positioned supine in bed. What important nursing assessment would be evaluated next? Compare the level of JVD on each side of the neck. Raise the head of the bed and evaluate the effect on the JVD. Have the client cough and assess the change in the JVD. Palpate the carotid arteries and assess for any changes in the JVD.

Raise the head of the bed and evaluate the effect on the JVD.

The nurse identifies which problem as most likely to lead to a sickle cell crisis? Recurrence of acute otitis media A fall with swelling at the kneecap and joint Fractured radius requiring internal fixation Recurrence of respiratory tract infection

Recurrence of respiratory tract infection

The nurse is assessing a client with venous insufficiency to the lower legs. What assessment finding does the nurse expect in this client? Select all that apply. Cold, bluish colored extremity Reddish-brown discoloration around ankles Appearance of an ulcer with irregular borders Absence of hair on the legs Diminished pedal and posterior tibial pulses Lower leg edema

Reddish-brown discoloration around ankles Appearance of an ulcer with irregular borders Lower leg edema

A 35-year-old male has enlarged lymph nodes in the neck and a mediastinal mass. He was diagnosed with Hodgkin lymphoma. Which of the following abnormal cells would be expected with this disease? Merkel cell Schwann cell Reed-Sternberg cell Kupffer cell

Reed-Sternberg cell

A client has had her blood pressure (BP) evaluated weekly for 1 month. At the end of the month, the nurse averages out the weekly blood pressures at 150/96 mm Hg. The client is 20 pounds (9.1 kg) overweight, and her cholesterol is 240 mg/dL (6.2 mmol/L). What is important information for the nurse to include in the teaching plan for this client? Refer her to the doctor for further follow-up and medications. Increase the fiber in her diet and begin a daily 30-minute workout. Reduce her sodium intake and decrease the dietary calories that come from fat. Reduce her cholesterol intake for 1 month and check her BP three times a week.

Refer her to the doctor for further follow-up and medications.

A client with hypovolemic shock has a urinary output of 30 mL/hr. Which explanation describes the compensatory physiologic mechanism that leads to altered urinary output? Alpha-adrenergic receptor stimulation, resulting in increased cardiac output with a compensatory increased heart rate and myocardial contractility Movement of interstitial fluid to the intravascular space, increasing renal blood flow Activation of the sympathetic nervous system, which leads to vasodilation of the renal arteries Release of aldosterone that increases serum osmolality, which leads to the release of antidiuretic hormone

Release of aldosterone that increases serum osmolality, which leads to the release of antidiuretic hormone

Which interventions are important in the care of the client with polycythemia vera? Select all that apply. Repeated phlebotomy to manage condition Administration of hydroxyurea Administration of interferon alpha Administration of hematinic agents Long-term prophylactic antibiotic therapy6Low-salt diet to decrease fluid volume

Repeated phlebotomy to manage condition Administration of hydroxyurea Administration of interferon alpha

Identify the order for administration of a unit of packed red blood cells (PRBCs). Put the steps in order from the first to the last step. Complete baseline vital signs. Start the blood transfusion. Monitor for signs and symptoms of blood transfusion reaction. Retrieve the blood from the blood bank. Request signature for blood transfusion consent. Blood is drawn and blood band applied.

Request signature for blood transfusion consent. Blood is drawn and blood band applied. Retrieve the blood from the blood bank. Complete baseline vital signs. Start the blood transfusion. Monitor for signs and symptoms of blood transfusion reaction.

An older adult client with multiple health issues, including hypothyroidism, hyperlipidemia, and type 2 diabetes, reports to the clinic with complaints of muscle aches, tenderness, and weakness in the lower extremities, since starting a new exercise program. The client has been taking simvastatin for the past 10 days. What serious complication would the nurse anticipate related to the medication? Vascular insufficiency Venous insufficiency Mild muscle injury to overuse Rhabdomyolysis

Rhabdomyolysis

The nurse is preparing to assess the client's central venous pressure. The physiologic zero point from which the measurement is referenced is located where on the client's chest? Third left costal cartridge, close to the sternum Right midclavicular line and the fifth intercostal space Right midaxillary line and the fourth intercostal space Area just below the junction of the sternum and the xiphoid

Right midaxillary line and the fourth intercostal space

A nurse observes jugular vein distention in a client with cardiac disease when his head is elevated 45 degrees. What condition will the nurse further assess for in this client? Dehydration Left-sided heart failure Right-sided heart failure Cardiovascular accident

Right-sided heart failure

What electrocardiogram (ECG) changes would reflect myocardial ischemia in a client who has been admitted for observation after experiencing an episode of chest pain? Prolonged PR interval Wide QRS complex ST-segment elevation or depression Tall, peaked T waves

ST-segment elevation or depression Myocardial ischemia is indicated by ST-segment elevation or depression, depending on the lead of the ECG. A STEMI (ST elevated MI) may have occurred. A prolonged PR interval is indicative of heart block, and a wide QRS complex can indicate a bundle branch block. Tall, peaked T waves occur as a result of hyperkalemia. If the tall T waves are seen throughout the ECG, hyperkalemia may be present.

The nurse would be correct in withholding a dose of digoxin in a client with systolic heart failure without specific instruction from the healthcare provider if which finding was documented? Serum digoxin level of 1.5 ng/mL (1.1 nmol/L) Blood pressure (BP) of 104/68 mm Hg Serum K of 2.5 mEq/L (2.5 mmol/L) Apical pulse of 68 beats/min

Serum K of 2.5 mEq/L (2.5 mmol/L)

An older adult client is taking 0.25 mg of digoxin (po) daily and 40 mg of furosemide (po) daily for heart failure. She has complained of increasing lethargy and nausea over the past 2 days; however, she is still able to take her medication. Her blood pressure is 150/98 mm Hg; pulse is 110 beats/min and irregular; respiratory rate is 18 breaths/min. What laboratory information is most important for the nurse to evaluate? Hemoglobin, hematocrit, and white blood cell count Arterial blood gases and acid-base balance Blood urea nitrogen (BUN) and serum creatinine levels Serum electrolyte levels, particularly potassium

Serum electrolyte levels, particularly potassium

A client in the emergency room is being admitted for acute angina. The nurse asks the client about medications he has been taking at home. Which medication will have implications on the client's immediate care? Sildenafil Regular insulin Propranolol Digoxin

Sildenafil

When performing a physical examination of the hematologic system, which systems are specifically relevant to the focused examination? Select all that apply. Skin Lymph nodes Spleen Liver Cardiac Respiratory Reproductive

Skin Lymph nodes Spleen Liver Cardiac Respiratory

A client diagnosed with peripheral vascular disease (PVD) is being discharged. Which of the client's risk factors would be most important for the nurse to discuss with the client before discharge? Alcohol intake Age Smoking Hypoglycemia

Smoking

The nurse is performing an initial assessment on a client recently diagnosed with thromboangiitis obliterans or Buerger's disease. Regarding the client's diagnosis, what information from the client's history would be of most concern? Chronic hypertension for 4 years Smoking a pack of cigarettes a day for 10 years Mother had Raynaud's disease History of type 2 diabetes

Smoking a pack of cigarettes a day for 10 years

A client has a significant number of cardiac risk factors, such as obesity, smoking, drinking alcohol daily, and working in a high stress computer job that is sedentary. The client tells the nurse he will work on one risk factor at a time. Which of the following should the nurse reinforce as the highest priority intervention to initially reduce risk? Weight reduction Starting an exercise program Eliminating alcohol consumption Smoking cessation

Smoking cessation

A client with peripheral artery disease (PAD) understands that the pain in his foot is caused by inadequate blood supply. The nurse would confirm which client activity as one that would further diminish blood flow to the extremities? Lowering the limb Drinking alcohol Smoking cigarettes Wearing antiembolism stockings

Smoking cigarettes

A patient prescribed warfarin sodium is instructed that certain foods decrease the effect of the drug. Which foods, if eaten, must be consumed in consistent amounts over the course of a week? Milk and cheese Spinach and asparagus Chicken and beef Fresh fruits

Spinach and asparagus

The nurse is caring for a client with venous blood pooling in the lower extremities caused by chronic venous insufficiency. The nurse would identify what assessment data that would correlate with this diagnosis? Select all that apply. Stasis dermatitis Diminished peripheral pulses Peripheral edema Gangrenous wounds Venous stasis ulcers Skin hyperpigmentation

Stasis dermatitis Peripheral edema Venous stasis ulcers Skin hyperpigmentation

A client begins to complain of chills and discomfort after about 50 mL of blood has transfused from a unit of packed red blood cells. What is the nurse's priority action? Slow down the infusing blood and dilute it with normal saline solution. Compare the vital signs now with what they were before the blood transfusion began. Stop the transfusion and maintain a patent line with normal saline solution and new tubing. Discontinue the transfusion, remove the intravenous (IV) catheter, and restart the transfusion in another site.

Stop the transfusion and maintain a patent line with normal saline solution and new tubing.

The nurse is caring for a client who is receiving a blood transfusion. The transfusion was started 30 minutes ago at a rate of 100 mL/hr. The client begins to complain of low back pain and headache and is increasingly restless. What is the first nursing action? Slow the infusion and evaluate the vital signs and the client's history of transfusion reactions. Stop the transfusion, disconnect the blood tubing, and begin a primary infusion of normal saline solution. Stop the infusion of blood and begin infusion of normal saline solution from the Y connector. Recheck the unit of blood for correct identification numbers and crossmatch information.

Stop the transfusion, disconnect the blood tubing, and begin a primary infusion of normal saline solution.

An infusion of packed RBC's is completed at 10 am. At 2 pm the client is experiencing problems the nurse suspects a transfusion reaction. What would be the important signs to watch for? Hypertension Pruritus and redness at infusion site Flushing and urticaria Sudden onset of dyspnea and hypoxia

Sudden onset of dyspnea and hypoxia

What would be important to include in the discharge plan that discusses monitoring for adverse effects with the long-term therapy with procainamide? Pulmonary stricture in bronchial tubes Slow, progressive increase in blood pressure Cinchonism with tinnitus and vertigo Systemic lupus erythematosus-like syndrome

Systemic lupus erythematosus-like syndrome

A client with hypertension is being treated with metoprolol. What are important nursing actions to perform prior to the first administration of the medication. Select all that apply. Take the apical pulse for 1 full minute. Check the blood pressure. Determine if urine output is 30 mL/hr. Review the client's electrocardiogram (ECG). Review the laboratory results for hypokalemia.

Take the apical pulse for 1 full minute. Check the blood pressure. Review the client's electrocardiogram (ECG).

Which of the following would be an appropriate nursing implication to teach the client in regard to the administration of cholestyramine? Supplement with water-soluble vitamins in long-term therapy. Mix powder with only 2 oz (60 mL) of fluid for administration. Teach the client to eat more fiber daily. Medication may be taken with meals.

Teach the client to eat more fiber daily.

The nurse is administering propranolol to a client who is being treated for hypertension. What is the desired response to this medication? Vasodilation occurs, resulting in a decrease in the cardiac afterload. The cardiac rate is decreased with a resulting decrease in the cardiac output. Cardiac output is decreased and the arterial blood pressure (BP) rises. Pericardial fluid is decreased, thus decreasing the cardiac workload.

The cardiac rate is decreased with a resulting decrease in the cardiac output.

The nurse applies a nitroglycerin patch on a client who has undergone cardiac surgery. What nursing observation indicates that the nitroglycerin patch is achieving the desired effect? The chest pain is completely relieved. The client performs activities of daily living without chest pain. The pain is controlled with frequent changes of the patch. The client tolerates increased activity without pain.

The client performs activities of daily living without chest pain.

A client comes into the emergency room (ER) via an ambulance and is complaining of severe chest pain. Which nursing assessment data would suggest the pain is caused by an acute coronary syndrome or myocardial infarction (MI)? The pain increases when the client inhales deeply. The client reports the pain has gotten progressively more severe over the past hour. The pain is immediately relieved with sublingual nitroglycerin. The pain is relieved by administering oxygen for 3 minutes via nasal cannula.

The client reports the pain has gotten progressively more severe over the past hour.

After morning report, the nurse has been assigned the following four clients. Which client would the nurse assess first? The client with type 1 diabetes with a draining ulcer on the right foot The client who is 1 hour postoperative left femoral atherectomy The client with intermittent claudication asking for pain medication The client on a nitroprusside drip for treatment of hypertensive crisis with a BP of 152/86 mm Hg

The client who is 1 hour postoperative left femoral atherectomy

A client is scheduled for a coronary angiography and possible percutaneous coronary intervention (PCI) after an episode of chest pain. What will the nurse explain to the client regarding the purpose of the procedure? The internal chambers will be visualized to determine if any structural damage has occurred. Obstructions in the coronary arteries will be found and a thrombolytic medication will be used to remove them. The amount of blood pumped from the heart will be measured to determine if it has decreased. The coronary arteries will be visualized, and if obstructions are found, they can be dilated.

The coronary arteries will be visualized, and if obstructions are found, they can be dilated.

A client has a permanent pacemaker inserted for treatment of a third-degree cardiac block. The client asks the nurse how the pacemaker will work. What will the nurse teach the client about the pacemaker? The pacemaker will identify the client's own heart beats and will stimulate a heartbeat if the client's heart rate drops too low. If ventricular tachycardia occurs, the pacemaker will identify the rhythm and take over the client's heart rate. If the client experiences a cardiac arrest, the pacemaker will take over and maintain adequate cardiac output. If the third-degree block worsens, the pacemaker will initiate the pacing of the heart rate.

The pacemaker will identify the client's own heart beats and will stimulate a heartbeat if the client's heart rate drops too low.

The nurse is assessing a client who has been admitted for evaluation of chronic stable angina. What assessment information would correlate with this diagnosis? Chest pain arouses the client around midnight to 3:00 am. The pain is relieved with one sublingual nitroglycerin tablet. The pain has been occurring more frequently over the past several weeks. The pain is primarily midsternal, radiating toward the left arm.

The pain is relieved with one sublingual nitroglycerin tablet.

The nurse is performing an assessment of a client who experienced an acute coronary syndrome (myocardial infarction [MI]) of the left ventricle 3 days ago. The nurse is concerned about the development of complications. On what area of the physical assessment would the nurse focus? The lower extremities and the jugular vein for the presence of systemic venous congestion The area on the chest where the point of maximum impulse (PMI) is heard most clearly The level of dependent edema and fluid intake over the past 24 hours The presence of dyspnea and auscultation of crackles in the lungs

The presence of dyspnea and auscultation of crackles in the lungs

The nurse is monitoring a cardiac client who is on a treadmill for an exercise stress test. After about 5 minutes on the treadmill what observation would be of immediate concern to the nurse? Client begins to complain of shortness of breath. Heart rate increases from 82 to 96 beats/min. Blood pressure increases from 140/80 to 158/90 mm Hg. There is an ST segment elevation from the baseline.

There is an ST segment elevation from the baseline.

What would be an important home care goal for a client who has infective endocarditis? To begin an exercise regimen as soon as possible To monitor urinary output To continue antibiotic therapy To decrease activity until pulse stabilizes

To continue antibiotic therapy

The nurse is preparing a client for cardiac catheterization. What is the best explanation regarding the purpose of a cardiac catheterization with coronary angiography? To study the conduction system To evaluate exercise tolerance To evaluate coronary artery blood flow To measure the pumping capacity of the heart

To evaluate coronary artery blood flow

What is the primary pathophysiologic reason for performing coronary artery bypass surgery? To increase the myocardial workload To increase oxygen supply to the heart muscle To reduce plaque buildup in the carotid artery To reduce overall contractility of the heart muscle

To increase oxygen supply to the heart muscle

While discussing her diagnosis of hypertension, a client asks the nurse how long she is going to have to take all of the medications that have been prescribed. On what principle is the nurse's response based? The client will be scheduled for an appointment in 2 months; the doctor will decrease her medications at that time. As soon as her blood pressure (BP) returns to normal levels, the client will be able to stop taking her medications. To maintain stable control of her BP, the client will have to take the medications indefinitely. The nurse cannot discuss the medications with the client; the client will need to talk with the doctor.

To maintain stable control of her BP, the client will have to take the medications indefinitely.

The patient is being treated for non-Hodgkin's lymphoma (NHL). What should the nurse first teach the patient about the treatment? Skin care that will be needed Method of obtaining the treatment Gastrointestinal tract effects of treatment Treatment type and expected side effects

Treatment type and expected side effects

The nurse understands that the pain of intermittent claudication increases when a client walks in the following situations. Select all that apply. Up an incline Fast In cold weather For an extended period In nonsupportive shoes Upstairs

Up an incline Fast For an extended period Upstairs

The nurse is assessing a client whose condition is being stabilized after experiencing an acute coronary syndrome (myocardial infarction). What finding on the nursing assessment would indicate inadequate renal perfusion? Decreasing serum blood urea nitrogen (BUN) level Urine specific gravity of less than 1.010 Urine output of less than 30 mL/hr Low urine osmolarity and creatinine clearance

Urine output of less than 30 mL/hr

What is an important nursing action in the safe administration of heparin? Check the prothrombin time (PT) and administer the medication if it is less than 20 seconds. Use a 20-gage, 1-inch (2.5 cm) needle and inject the medication into the deltoid muscle and gently massage the area. Dilute the medication in 50 mL of D5W and infuse by intravenous piggyback (IVPB) over 15 minutes. Use a 25-gage, ½-inch (1.3 cm) needle and inject the medication into the subcutaneous tissue of the abdomen.

Use a 25-gage, ½-inch (1.3 cm) needle and inject the medication into the subcutaneous tissue of the abdomen.

The nurse is instructing a patient and caregiver on warfarin therapy at home. Which items does the nurse include in the teaching plan? (Select all that apply.) Use electric razors, not straight razors. Take a stool softeners to avoid hard stools and straining. Use soft toothbrushes for oral care. If you cut yourself hold manual pressure for 2 minutes so the cut can clot. Wear tight fitting clothing.

Use electric razors, not straight razors. Take a stool softeners to avoid hard stools and straining. Use soft toothbrushes for oral care.

The nurse is teaching the client information about an intranasal preparation of vitamin B12. What would be appropriate to explain to the client? Before administering it, have a hot cup of tea or coffee. Dilute the medication with saline solution before taking it. Use the metered-dose inhaler that comes with the medication. Avoid hot foods for 10 minutes after administering the medication.

Use the metered-dose inhaler that comes with the medication.

A transesophageal echocardiogram (TEE) is ordered for evaluation of a client with a possible mitral valve stenosis. What is a priority nursing action before the beginning of the procedure? Validate that the client has taken nothing by mouth (NPO) for the past 6 to 8 hours. Begin oxygen at 3 L/min via nasal cannula. Assess that the preprocedure preparation has been initiated. Administer lorazepam via intravenously (IV).

Validate that the client has taken nothing by mouth (NPO) for the past 6 to 8 hours.

A client has a history of atherosclerotic heart disease with a sustained increase in blood pressure. What side effect may occur should the client use an over-the-counter decongestant? Urinary frequency and diuresis Bradycardia and diarrhea Vasoconstriction and increased arterial pressure Headache and dysrhythmias

Vasoconstriction and increased arterial pressure

A client who is diagnosed with pernicious anemia and is experiencing malabsorption of oral B12 is going to begin self-administration of vitamin B12 injections. In which muscle should the nurse teach the client to inject the vitamin B12? Abdominal Gluteus maximus Deltoid Vastus lateralis

Vastus lateralis

The nurse is evaluating a cardiac client's progress. What information would be indicative of cardiac compensatory mechanisms? Headache, drowsiness, confusion Bradycardia, restlessness, hyperventilation Ventricular dilation, hypertrophy, tachycardia Hepatomegaly, splenomegaly, cardiac hypertrophy

Ventricular dilation, hypertrophy, tachycardia

The nurse is preparing a client for a cardiac catheterization. Which nursing interventions are necessary in preparing the client for this procedure? Select all that apply. Verify that the consent form has been signed. Explain the procedure to the client. Provide a clear liquid, no caffeine diet. Evaluate peripheral pulses. Obtain a 12-lead electrocardiogram (ECG). Obtain a history for shellfish allergy.

Verify that the consent form has been signed. Evaluate peripheral pulses. Obtain a history for shellfish allergy.

A client arrives at the emergency room (ER) complaining that she has had several episodes of epistaxis. The client also states that she is taking warfarin, and the client's stools test positive for the presence of blood. What medication would the nurse anticipate administering? Vitamin K Protamine sulfate Ascorbic acid Calcium chloride

Vitamin K

The nurse is preparing to administer the following medications to a client who has been hospitalized with a history of deep vein thrombosis. Which medication is most important to double-check with another licensed nurse? Nizatidine 20 mg intravenous (IV) Furosemide 40 mg IV Warfarin 2.5 mg by mouth (po) Digoxin 0.25 mg po

Warfarin 2.5 mg by mouth (po)

The nurse is preparing a client for a cardiac catheterization to determine status of the coronary arteries. What information would be important for the nurse to provide this client about the test? A catheter will be inserted into the left femoral vein and directed toward the heart. You will be required to be on bed rest with cardiac monitoring for 12 hours after the test. You may feel short bursts of chest pain as the catheter is progressed. When the contrast material is injected, you may experience a warm, flushed feeling.

When the contrast material is injected, you may experience a warm, flushed feeling.

The nurse is teaching a 79-year-old client about a new medication that has been ordered by the healthcare provider. What information should the nurse include in teaching the client about enalapril? Select all that apply. Will need to have renal function monitored. May have first-dose effect. Monitor pulse daily. Obtain a baseline liver enzyme panel. Report any persistent cough.

Will need to have renal function monitored. May have first-dose effect. Report any persistent cough.

A nurse is preparing to administer morning medications to a client with heart failure (HF), including digoxin. The client's apical pulse is 54. What is the nurse's priority action in regard to administering this medication? Document the finding and administer the medication. Administer the medication and notify the healthcare provider. Administer the medication and recheck the client's pulse rate in 30 minutes. Withhold the medication and notify the healthcare provider.

Withhold the medication and notify the healthcare provider.

The nurse notes that a client who is receiving verapamil for atrial flutter has a heart rate of 60 beats/min and a blood pressure of 108/68 mm Hg. What would be the appropriate action as the nurse prepares the next dose? Withhold the medication and notify the healthcare provider. Administer the next scheduled dose. After administering the dose, recheck vital signs in an hour. Administer intravenous (IV) epinephrine.

Withhold the medication and notify the healthcare provider.

Which of the following findings is indicative of accentuated left ventricular filling in a patient with chronic mitral regurgitation? an audible third heart sound and a late diastolic murmur a midsystolic click followed by an early systolic murmur an audible third heart sound and a pansystolic or holosystolic murmur an audible third heart sound and a middiastolic click with a late diastolic murmur

an audible third heart sound and a pansystolic or holosystolic murmur

Prophylactic antibiotics are indicated to prevent infective endocarditis for at-risk individuals who are undergoing any dental procedure. are entering the third trimester of pregnancy. have acquired a viral respiratory tract infection. are exposed to human immunodeficiency virus.

are undergoing any dental procedure.

The nurse hears the patient state to a family member, "My ears are ringing all the time." The nurse assesses the patient and reviews the medication record for drugs with the side effect of tinnitus. Which medication(s) is the nurse expecting to find on the medication administration record? Select all that apply. aspirin furosemide methyldopa propranolol nitroglycerin

aspirin furosemide

The nurse would suspect disseminated intravascular coagulation (DIC) if the client's laboratory results indicated increased prothrombin time, decreased platelet count, and decreased fibrinogen level. decreased prothrombin time, increased hematocrit, and increased fibrinogen level. increased platelet count, decreased hematocrit, and increased prothrombin time. increased platelet count, increased hematocrit, and decreased prothrombin time.

increased prothrombin time, decreased platelet count, and decreased fibrinogen level.

Penicillin therapy for the patient with rheumatic fever is indicated to prevent chronic rheumatic carditis. relieve arthralgia and inflamed joints. prevent reinfection and recurrent rheumatic fever. destroy the infective microorganism and cure the disease.

prevent reinfection and recurrent rheumatic fever.

The nurse suspects cardiac tamponade in a patient with acute pericarditis based on the finding of chest pain. pulsus paradoxus. mitral valve murmur. pericardial friction rub.

pulsus paradoxus.

The most common cause of mitral valve stenosis is myocarditis. rheumatic heart disease. congenital heart disease. subacute infective endocarditis.

rheumatic heart disease.

The nurse plans care for the patient with hypertrophic cardiomyopathy based on the knowledge that family members may be at risk because of the infectious nature of the disease. medical management of the disorder focuses on treatment of the underlying cause. the prognosis of the patient is poor, and emotional support is a high priority of care. the condition may be successfully treated with surgical ventriculomyotomy and myectomy.

the prognosis of the patient is poor, and emotional support is a high priority of care.

The nurse teaches a group of patients diagnosed with hypertension that influencing factors related to rising blood pressure are associated with which physiological processes? Select all that apply. vasoconstriction increased blood viscosity increased cardiac output increased blood volume dehydration dysrhythmia

vasoconstriction increased blood viscosity increased cardiac output increased blood volume


Kaugnay na mga set ng pag-aaral

Intro to Supply chain Exam 2 Rutgers

View Set

Human growth and development quiz questions

View Set

Principles of Biblical Studies 1 FINAL

View Set

U.S. History - Ch. 16, Sec. 2 - Questions

View Set

Social Psychology Quiz 1: Practice Problems

View Set